Как найти аппликату точки по координатам

Рис. 1

Аппликатой точки A называется координата этой точки на оси OZ в прямоугольной трёхмерной системе координат. Величина аппликаты точки A равна длине отрезка OD (см. рис. 1). Если точка D принадлежит положительной полуоси OZ, то аппликата имеет положительное значение. Если точка D принадлежит отрицательной полуоси OZ, то аппликата имеет отрицательное значение. Если точка A лежит на плоскости XOY, то её аппликата равна нулю.

Слово «аппликата» происходит от лат. applicata, что означает «приложенная». Имеется в виду, что координата Z (аппликата) была приложена к уже имевшимся двум координатам на плоскости: абсциссе и ординате.

В прямоугольной системе координат ось OZ называется «осью аппликат».

См. также

  • Ордината
  • Абсцисса

Содержание:

Система координат в пространстве

Декартова система координат в пространстве

Вы познакомились с декартовой системой координат на плоскости в предыдущих классах. Систему координат в пространстве введём аналогично тому, как это было сделано на плоскости. Рассмотрим три взаимно перпендикулярных оси Ох, Оу и Оz, пересекающихся в точке О, являющейся началом координат. Через каждую пару этих прямых проведём плоскости Оху, 0xz и Оуz (рис. 1). Таким образом вводится система координат в пространстве, при этом

точку О — называют началом координат, прямые Ох, Оу и Оzосями координат, Охось абсцисс, Оу ось ординат и Оzось аппликат, плоскости Оху, Оуz и Охzкоординатными плоскостями.

Система координат в пространстве - определение с примерами решения

Координатные плоскости делят пространство на 8 октант (получетвертей) (рис. 1).

Пусть в пространстве задана произвольная точка А. Через эту точку проведём плоскости, перпендикулярные плоскостям Охz, Оуz и Охz (рис. 2). Одна из этих плоскостей пересечёт ось Ох в точке Ах.

Координату Ах на оси Ох называют координатой х или абсциссой точки А.

Аналогично определяют у — координату (ординату) и z- координату (аппликату) точки А.

Координаты точки А записывают в виде А (х; у; z) или короче (х; у; z). Точки, изображённые на рисунке 3, имеют следующие координаты: А (0; 5; 0), B (4; 0; 0), М (0; 5; 4), К (2; 3; 4), Р (-2; 3; -4). Система координат в пространстве - определение с примерами решения

Пример:

Пусть в пространстве в декартовой системе координат

задана точка А (2; 3; 4). Где она расположена?

Решение:

От начала координат в положительном направлении осей Ох и Оу отложим отрезки ОАх = 2 и ОАу = 3 (рис. 4).

Через точку Ах проведём прямую, лежащую в плоскости Оху и параллельную оси Оу. А через точку Аy проведём прямую, лежащую в плоскости Оху и параллельную оси Ох. Точку пересечения этих прямых обозначим A1 . Через точку A1 проведём прямую, перпендикулярную плоскости Оху и на ней в положительном направлении Oz отложим отрезок АА1 = 4. Тогда точка А (2; 3; 4) и будет искомой точкой. Система координат в пространстве - определение с примерами решения

Пользуясь системой координат, созданной для современных программируемых станков и автоматизированных роботов, составляются программы, на основе которых обрабатываются металлы (рис. 5).

Система координат в пространстве - определение с примерами решения

Расстояние между двумя точками

Пусть заданы две точки А (х1; у1; z1) и B (х2; у2; z2).

1.Сначала рассмотрим случай, когда прямая АВ не параллельна оси Оz (рис. 6). Через точки А и В проведём прямые, параллельные оси Оz. И пусть они пересекают плоскость Оху в точках Аz и Вz .

Координаты х и у этих точек соответственно равны координатам х и у точек А, В, а координаты z равны 0.

Теперь через точку В проведём плоскость а, параллельную плоскости Оху. Она пересечёт прямую ААz в некоторой точке С.

По теореме Пифагора: АВ2 = АС2 + СВ2.

Однако Система координат в пространстве - определение с примерами решения

Поэтому Система координат в пространстве - определение с примерами решения

2.Пусть отрезок АВ параллелен оси Оz, тогда Система координат в пространстве - определение с примерами решения и, так как

х1= х2 , у1 = у2 , мы опять приходим к вышеприведённой формуле.

Следовательно, расстояние между двумя точками А и В:

Система координат в пространстве - определение с примерами решения (1)

Примечание. Формула (1) выражает длину диагонали прямоугольного параллелепипеда, измерения которого равны Система координат в пространстве - определение с примерами решения

Уравнение сферы и шара

Известно, что множество всех точек М (х; у; z), расположенных на расстоянии R от данной точки А (а; Ь; с) образуют сферу (рис. 7). Тогда по формуле (1) координаты всех точек, расположенных на сфере радиуса R с центром в точке А (а; b; с), удовлетворяют равенству Система координат в пространстве - определение с примерами решения

Отсюда, ясно, что неравенство для точек шара радиуса R с центром в

точке А (а; b; с) имеет вид: Система координат в пространстве - определение с примерами решения

Система координат в пространстве - определение с примерами решения

Пример:

Найдите периметр треугольника ABC с вершинами в

точках А (9; 3; -5), В (2; 10; -5), С (2; 3; 2).

Решение:

Р=АВ+АС+ВС периметр треугольника ABC. Воспользовавшись формулой Система координат в пространстве - определение с примерами решения расстояния между двумя точками, найдём длины сторон треугольника:

Система координат в пространстве - определение с примерами решения

Следовательно, треугольник ABC равносторонний и его периметр Система координат в пространстве - определение с примерами решения.

Ответ: Система координат в пространстве - определение с примерами решения

Координаты середины отрезка

Пусть А (x1; y1;z1) и В (х2; у2; z2) — произвольные точки, точка С (х; у; z) середина отрезка AB (рис. 8). Система координат в пространстве - определение с примерами решения

Через точки А, В и С проведём прямые, параллельные оси пересекающие плоскость Оху в точках Система координат в пространстве - определение с примерами решения и Система координат в пространстве - определение с примерами решения. Тогда по теореме Фалеса точка Сz — середина отрезка АzВz.

Отсюда по формулам нахождения координат середины отрезка на плоскости Система координат в пространстве - определение с примерами решения

Чтобы найти координату z, нужно вместо плоскости Оху рассмотреть плоскость 0xz или Оуz.

Тогда и для z получим формулу, подобную вышеприведённой.

Система координат в пространстве - определение с примерами решения

Аналогично, используя координаты концов A и B отрезка AB, по формулам Система координат в пространстве - определение с примерами решения

находят координаты точки Р(х1;у]; г,), делящей отрезок АВ в отношении X САР: РВ = X).

Задача 3. Докажите, что четырёхугольник МЛШЬ с вершинами М{3; 6; 4), N(0; 2; 4), К(3; 2; 8), 1(6; 6; 8) — параллелограмм (рис. 9).

Доказательство: Для решения задачи используем признак параллелограмма: Четырёхугольник, точка пересечения диагоналей которого делит их пополам, является параллелограммом.

Координаты середины отрезка МК:

Система координат в пространстве - определение с примерами решения

Координаты середины отрезка NL:

Система координат в пространстве - определение с примерами решения

Координаты середин отрезков МК и NL равны. Это говорит о том, что отрезки пeрeсeкаются и в точке пeрeсeчeния делятся пополам. Следовательно, четырёхугольник MNLK — параллелограмм.Система координат в пространстве - определение с примерами решения

В переписке с известным целителем и математиком Абу Али ибн Сино Абу Райхон Беруни задаёт следующий вопрос: «Почему Аристотель и другие (философы) называют шесть сторон?»

Рассматривая шестисторонний куб, Беруни говорит о фигурах «с другим количеством сторон» и добавляет, что «шарообразные фигуры не имеют сторон.» А Ибн Сино отвечает, что «во всех случаях нужно считать, что сторон шесть, так как у каждой фигуры, независимо от её формы, есть три измерения — длина, глубина и ширина».

Здесь Ибн Сино имеет ввиду три координаты, именуемые условно «шесть сторон».

В произведении «Канон Масъуда» Беруни приводит точное математическое определение шести сторон: «Сторон шесть, так как они ограничивают движение фигур по своим измерениям. Измерений три: длина, ширина и глубина. А их в два раза больше самих измерений.»

В предыдущих книгах автор определяет положение небесных тел с помощью двух координат относительно небесной сферы — эклиптического уравнения. Либо через те же координаты, но относительно небесного экватора или горизонта. Однако при определении взаимного расположения звёзд и небесных светил придётся учитывать и случаи затмений. Вот в таких случаях появляется необходимость в третьей сферической координате. Эта необходимость привела Беруни к отказу от теории небесных координат.

Векторы в пространстве и действия над ними

Векторы в пространстве

Понятие вектора в пространстве вводят также как на плоскости.

Вектором в пространстве называют направленный отрезок. Основные понятия, относящиеся к векторам в пространстве, аналогичны этим понятиям на плоскости: длина (модуль), направление вектора, равенство векторов.

Система координат в пространстве - определение с примерами решения

Координатами вектора с началом в точке А (х1; у1; z1) и концом в точке В (х1; у1; z1) называют числа Система координат в пространстве - определение с примерами решения, (рис. 17).

Приведем без доказательства свойства векторов, аналогичных свойствам на плоскости.

Также как на плоскости, соответствующие координаты равных векторов равны и, обратно, векторы с равными координатами равны.

Hа основании этого вектор можно обозначить как Система координат в пространстве - определение с примерами решения или Система координат в пространстве - определение с примерами решения или кратко Система координат в пространстве - определение с примерами решения (рис. 18).

Вектор можно записать и без координат Система координат в пространстве - определение с примерами решения (или Система координат в пространстве - определение с примерами решения). В этой записи

на первом месте начало вектора, а на втором — конец.

Вектор с координатами, равными нулю, называют нулевым вектором и обозначают Система координат в пространстве - определение с примерами решения или Система координат в пространстве - определение с примерами решения, направление этого вектора не определено.

Если начало вектора расположено в начале координат О, а числа а1,

а2 и а3 — координаты точки А, то есть А (а1; а2; а3), то эти же числа будут

координатами вектора Система координат в пространстве - определение с примерами решения: Система координат в пространстве - определение с примерами решения (а1; а2; а3).

Однако вектор в пространстве Система координат в пространстве - определение с примерами решения с началом в точке К(с1; с2; с3) и концом в точке Система координат в пространстве - определение с примерами решения будет иметь те же координаты: Система координат в пространстве - определение с примерами решения.

Отсюда следует, что вектор можно приложить к любой точке пространства. В геометрии мы рассматриваем такие свободные векторы. Но в физике, обычно вектор связан с некоторой точкой. Например, воздействие силы приложенная к пружине F на рисунке 19 зависит от точки её приложения.

Длинной вектора называют длину направленного отрезка

изображающего его (рис. 17). Длину вектора Система координат в пространстве - определение с примерами решения записывают

такСистема координат в пространстве - определение с примерами решения. Длина вектора Система координат в пространстве - определение с примерами решения, заданного координатами,

вычисляется по формуле Система координат в пространстве - определение с примерами решения .

Пример:

Даны точки А (2; 7;-3),В (1; 0; 3), С (-3;-4; 5) и D (-2; 3; -1). Какие из векторов Система координат в пространстве - определение с примерами решенияи Система координат в пространстве - определение с примерами решения равны между собой?

Решение:

У равных векторов равны соответствующие координаты. Поэтому найдём координаты векторов:

Система координат в пространстве - определение с примерами решения

Следовательно, Система координат в пространстве - определение с примерами решения.

Докажите самостоятельно, что Система координат в пространстве - определение с примерами решения

Действия над векторами в пространстве

Действия над векторами. Сложение векторов, умножение на число и их скалярное произведение определяется также как на плоскости.

Суммой векторов Система координат в пространстве - определение с примерами решения и Система координат в пространстве - определение с примерами решения (b1; b2; b3); называют вектор Система координат в пространстве - определение с примерами решения (рис. 20).

Система координат в пространстве - определение с примерами решения

Пусть кран на рисунке 20.b движется вдоль вектора Система координат в пространстве - определение с примерами решения, а груз относительно крана вдоль вектора Система координат в пространстве - определение с примерами решения. В результате груз движется вдоль вектора Система координат в пространстве - определение с примерами решения. Поэтому из рисунка 20.с, на котором изображён сюжeт басни русского писателя И.А.Крылова, ясно, что герои басни не смогут сдвинуть телегу с места.

Свойства суммы векторов

Для любых векторов Система координат в пространстве - определение с примерами решения,Система координат в пространстве - определение с примерами решения и Система координат в пространстве - определение с примерами решения имеют место следующие свойства:

a)Система координат в пространстве - определение с примерами решения — переместительный закон сложения векторов;

b)Система координат в пространстве - определение с примерами решения — распределительный закон сложения.

Правило треугольника сложения векторов

Для любых точек А, В и С (рис. 21): Система координат в пространстве - определение с примерами решения

Правило параллелограмма сложения векторов

Если АВСD — параллелограмм (рис. 22), то Система координат в пространстве - определение с примерами решения

Правило многоугольника сложения векторов

Если точки А, В, С, D и Е — вершины многоугольника (рис. 23), тоСистема координат в пространстве - определение с примерами решения

Система координат в пространстве - определение с примерами решения

Правило параллелепипеда сложения трёх векторов, не лежащих в одной плоскости. Если АВСDА1В1С1D1 параллелепипед (рис. 24), то

Система координат в пространстве - определение с примерами решения.

Вектор Система координат в пространстве - определение с примерами решенияСистема координат в пространстве - определение с примерами решения​​​​​​= (Система координат в пространстве - определение с примерами решенияa1; Система координат в пространстве - определение с примерами решенияa2; Система координат в пространстве - определение с примерами решенияa3) — называют умножением вектора

Система координат в пространстве - определение с примерами решения (a1; a2; a3) на число Система координат в пространстве - определение с примерами решения (рис. 25). Свойства операции умножения вектора на число.

Для любых векторов Система координат в пространстве - определение с примерами решения и Система координат в пространстве - определение с примерами решения и чисел Система координат в пространстве - определение с примерами решения и Система координат в пространстве - определение с примерами решения

а)Система координат в пространстве - определение с примерами решения;

b)Система координат в пространстве - определение с примерами решения;

c)Система координат в пространстве - определение с примерами решения и направление вектора Система координат в пространстве - определение с примерами решенияСистема координат в пространстве - определение с примерами решения

совпадает с направлением вектора Система координат в пространстве - определение с примерами решения, если Система координат в пространстве - определение с примерами решения,

противоположно направлению вектора Система координат в пространстве - определение с примерами решения, если Система координат в пространстве - определение с примерами решения. Система координат в пространстве - определение с примерами решения

Коллинеарные и компланарные векторы

Пусть заданы ненулевые векторы Система координат в пространстве - определение с примерами решения и Система координат в пространстве - определение с примерами решения. Если векторы

Система координат в пространстве - определение с примерами решения и Система координат в пространстве - определение с примерами решения сонаправлены или противоположно направлены,

то их называют коллинеарными векторами (рис. 26).

Свойство 1. Если для векторов Система координат в пространстве - определение с примерами решения и Система координат в пространстве - определение с примерами решения имеет место равенство Система координат в пространстве - определение с примерами решения, то они коллинеарны и наоборот.

Если Система координат в пространстве - определение с примерами решения, то векторы Система координат в пространстве - определение с примерами решения и Система координат в пространстве - определение с примерами решения сонаправлены Система координат в пространстве - определение с примерами решения, еслиСистема координат в пространстве - определение с примерами решения, то

противоположно направлены Система координат в пространстве - определение с примерами решения.

Свойство 2. Если векторы Система координат в пространстве - определение с примерами решения (a1; a2; a3) и Система координат в пространстве - определение с примерами решения (b1; b2; b3) коллинеарны,

то их соответствующие координаты пропорциональны:

Система координат в пространстве - определение с примерами решения и наоборот.

Пример:

Найдите вектор с началом в точке А (1; 1; 1) и концом в точке В, лежащей в плоскости Оху, коллинеарный вектору Система координат в пространстве - определение с примерами решения( 1; 2; 3).

Решение:

Пусть точка В имеет координаты В (х; у; z). Так как точка В лежит в плоскости Оху, то z=0. Тогда Система координат в пространстве - определение с примерами решения(х — 1 ;у — 1; — 1).

По условию задачи векторы Система координат в пространстве - определение с примерами решения(х — 1 ;у — 1; — 1) и Система координат в пространстве - определение с примерами решения(1, 2, 3) коллинеарны. Следовательно, их координаты пропорциональны.

Тогда получаем следующие пропорции Система координат в пространстве - определение с примерами решения.

Откуда находим Система координат в пространстве - определение с примерами решения, Система координат в пространстве - определение с примерами решения.

Итак,Система координат в пространстве - определение с примерами решения

Векторы, лежащие в одной плоскости или параллельных плоскостях, называют компланарными векторами (рис. 27). Система координат в пространстве - определение с примерами решения

Векторы Система координат в пространстве - определение с примерами решения(1; 0; 0), Система координат в пространстве - определение с примерами решения(0; 1; 0) и Система координат в пространстве - определение с примерами решения(0; 0; 1) называют ортами (рис. 28).

Любой вектор Система координат в пространстве - определение с примерами решения можно единственным образом разложить по ортам, то есть представить в виде Система координат в пространстве - определение с примерами решения(рис. 29).

Система координат в пространстве - определение с примерами решения

Точно также, если заданы три нeкомпланарных вектора Система координат в пространстве - определение с примерами решения и Система координат в пространстве - определение с примерами решения, то любой вектор Система координат в пространстве - определение с примерами решения можно единственным образом представить в виде:

Система координат в пространстве - определение с примерами решения.

Здесь Система координат в пространстве - определение с примерами решения некоторые действительные числа. Тогда говорят, что вектор разложен по заданным векторам.

Скалярное произведение векторов

Углом между ненулевыми векторами Система координат в пространстве - определение с примерами решения и Система координат в пространстве - определение с примерами решения называют угол между направленными отрезками векторов Система координат в пространстве - определение с примерами решения = Система координат в пространстве - определение с примерами решения и Система координат в пространстве - определение с примерами решения=Система координат в пространстве - определение с примерами решения, исходящих из точки О (рис. 30).

Угол между векторами Система координат в пространстве - определение с примерами решения и Система координат в пространстве - определение с примерами решения обозначают так Система координат в пространстве - определение с примерами решения.

Система координат в пространстве - определение с примерами решения

Скалярным произведением векторов Система координат в пространстве - определение с примерами решения и Система координат в пространстве - определение с примерами решения называют произведение длин этих векторов на косинус угла между ними.

Если один из векторов нулевой, то скалярное произведение этих векторов равно нулю.

Скалярное произведение обозначают Система координат в пространстве - определение с примерами решения или Система координат в пространстве - определение с примерами решения. По определениюСистема координат в пространстве - определение с примерами решения (1)

Из определения следует, что если скалярное произведение векторов Система координат в пространстве - определение с примерами решения и Система координат в пространстве - определение с примерами решения равно нулю, то эти векторы перпендикулярны и наоборот.

В физике работа A, выполненная при движении тела на расстоянии Система координат в пространстве - определение с примерами решения, под воздействием силы Система координат в пространстве - определение с примерами решения (рис. 31), равна скалярному произведению силы Система координат в пространстве - определение с примерами решенияна расстояниеСистема координат в пространстве - определение с примерами решения: Система координат в пространстве - определение с примерами решения

Свойство. Если Система координат в пространстве - определение с примерами решения и Система координат в пространстве - определение с примерами решения (b1; b2; b3), то (Система координат в пространстве - определение с примерами решенияСистема координат в пространстве - определение с примерами решения) = Система координат в пространстве - определение с примерами решения

Доказательство. Приложим векторы Система координат в пространстве - определение с примерами решения и Система координат в пространстве - определение с примерами решения к началу

координат О (рис.32). Тогда Система координат в пространстве - определение с примерами решения= Система координат в пространстве - определение с примерами решения и Система координат в пространстве - определение с примерами решения= (b1; b2; b3).

Если векторы неколлинеарны, то получаем треугольник АВО , для которого справедлива теорема косинусов.

Система координат в пространстве - определение с примерами решения

ТогдаСистема координат в пространстве - определение с примерами решения .

Однако, Система координат в пространстве - определение с примерами решения,Система координат в пространстве - определение с примерами решения

и Система координат в пространстве - определение с примерами решения.

Следовательно,Система координат в пространстве - определение с примерами решения

Система координат в пространстве - определение с примерами решения

Система координат в пространстве - определение с примерами решения.

Самостоятельно докажите, что и в случае, когда данные векторы коллинеарны Система координат в пространстве - определение с примерами решения, также выполняется

это равенство. Система координат в пространстве - определение с примерами решения

Свойства скалярного произведения векторов

1.Система координат в пространстве - определение с примерами решения — переместительное свойство.

2.Система координат в пространстве - определение с примерами решения — распределительное свойство.

3.Система координат в пространстве - определение с примерами решения — сочетательное свойство.

4.Если векторы а и b являются сонаправленными коллинеарными

векторами, то Система координат в пространстве - определение с примерами решения, так как соs 0° = 1.

5.Если же векторы противоположно направлены, то Система координат в пространстве - определение с примерами решения, так как cos l80° = -1.

6. Система координат в пространстве - определение с примерами решения.

7. Если векторСистема координат в пространстве - определение с примерами решения перпендикулярен вектору Система координат в пространстве - определение с примерами решения, то Система координат в пространстве - определение с примерами решения. Следствия: а) Длина вектора Система координат в пространстве - определение с примерами решения ; (1) b) косинус угла между векторами

Система координат в пространстве - определение с примерами решения : Система координат в пространстве - определение с примерами решения; (2)

с) условие перпендикулярности векторов Система координат в пространстве - определение с примерами решения и

Система координат в пространстве - определение с примерами решения.

Система координат в пространстве - определение с примерами решения (3)

Пример:

Система координат в пространстве - определение с примерами решения — заданные точки. Найдите косинус угла между векторами Система координат в пространстве - определение с примерами решения.

Решение:

Найдём длины векторов Система координат в пространстве - определение с примерами решения:

Система координат в пространстве - определение с примерами решения,

Система координат в пространстве - определение с примерами решения.

Система координат в пространстве - определение с примерами решения ,

Система координат в пространстве - определение с примерами решения .

Следовательно,

Система координат в пространстве - определение с примерами решения

Пример:

Найдите угол между векторами Система координат в пространстве - определение с примерами решения.

Решение:

Система координат в пространстве - определение с примерами решения Итак, Система координат в пространстве - определение с примерами решения

Пример:

Найдите Система координат в пространстве - определение с примерами решения, если Система координат в пространстве - определение с примерами решения, Система координат в пространстве - определение с примерами решения и угол между векторамиСистема координат в пространстве - определение с примерами решенияи Система координат в пространстве - определение с примерами решения равен Система координат в пространстве - определение с примерами решения .

Решение:

Система координат в пространстве - определение с примерами решения

Система координат в пространстве - определение с примерами решения

Пример:

Найдите координаты и длины векторов 1)Система координат в пространстве - определение с примерами решения; 2)Система координат в пространстве - определение с примерами решения, если Система координат в пространстве - определение с примерами решения.

Решение:

Подставим в выражения искомых векторов разложения векторов Система координат в пространстве - определение с примерами решенияи Система координат в пространстве - определение с примерами решения по координатам:

1)Система координат в пространстве - определение с примерами решения

Система координат в пространстве - определение с примерами решения. Следовательно,Система координат в пространстве - определение с примерами решения.

ТогдаСистема координат в пространстве - определение с примерами решения.

2)Система координат в пространстве - определение с примерами решения

Система координат в пространстве - определение с примерами решения Система координат в пространстве - определение с примерами решения.

Следовательно, Система координат в пространстве - определение с примерами решения.

Тогда Система координат в пространстве - определение с примерами решения

  • Заказать решение задач по высшей математике

Пример:

Найдите произведениеСистема координат в пространстве - определение с примерами решения, если угол между векторами Система координат в пространстве - определение с примерами решенияи Система координат в пространстве - определение с примерами решения равен 30° и Система координат в пространстве - определение с примерами решения , Система координат в пространстве - определение с примерами решения.

Решение:

Сначала найдём поизведение векторов Система координат в пространстве - определение с примерами решенияи Система координат в пространстве - определение с примерами решения :

Система координат в пространстве - определение с примерами решения.

Затем перемножим заданные выражения как многочлены

и, пользуясь распределительным свойством умножения

вектора на число, получим:

Система координат в пространстве - определение с примерами решения

Система координат в пространстве - определение с примерами решения.

Учитывая, что Система координат в пространстве - определение с примерами решения,

Система координат в пространстве - определение с примерами решения найдём искомое произведение

Система координат в пространстве - определение с примерами решения

Преобразование и подобие в пространстве

Геометрические преобразования в пространстве

Если каждую точку заданной в пространстве фигуры F изменить одним и тем же способом, то получим фигуру F1. Если при этом преобразовании различные точки первой фигуры переходят в различные точки второй, то говорят о преобразовании геометрической фигуры.

Если рассматривать все пространства как геометрическую фигуру, то также можно говорить о преобразовании геометрической фигуры.

Понятие геометрического преобразование в пространстве вводят также как на плоскости. Следовательно, свойства некоторых рассматриваeмых ниже видов преобразований и их доказательства также подобны соответствующим им на плоскости. Поэтому, мы не будем доказывать их и рекомендуем провести их самостоятельно.

Движение и параллельный перенос

Преобразование фигур, при котором сохраняются расстояния между точками, называют движением. Можно привести следующие свойства движения. При движении прямая переходит в прямую, луч — в луч, отрезок — в равный ему отрезок, угол — в равный ему угол, треугольник — в равный ему треугольник, плоскость — в плоскость, тетраэдр — в равный ему тетраэдр.

В пространстве фигуры, которые можно перевести одну в другую при некотором движении называют равными фигурами.

Простейшим примером движения является параллельный перенос.

Система координат в пространстве - определение с примерами решения

Пусть в пространстве даны векторСистема координат в пространстве - определение с примерами решения и произвольная точка Х

(рис. 44). Говорят, что точка Х перешла в точку X1 параллельным

переносом на вектор Система координат в пространстве - определение с примерами решения, если выполняется условие Система координат в пространстве - определение с примерами решения. Если каждую точку фигуры F сдвинуть на вектор Система координат в пространстве - определение с примерами решения при помощи параллельного переноса (рис. 45), то получим фигуру F1. Тогда говорят, что фигура F получена параллельным переносом фигуры F1 . При параллельном переносе каждая точка фигуры F сдвигается в одном и том же направлении на одно и то же расстояние.

Каждая точка подъёмного крана, изображённого на рисунке 46, параллельно перенесена на 40 м относительно начального положения.

Ясно, что параллельный перенос является движением. Поэтому прямая переходит в прямую, луч — в луч, плоскость — в плоскость,

и т. д.

Пусть точка Система координат в пространстве - определение с примерами решения фигуры F перешла в точку Система координат в пространстве - определение с примерами решения

фигуры F1 при помощи параллельного переноса

на вектор Система координат в пространстве - определение с примерами решения.

Тогда по определению получим:

Система координат в пространстве - определение с примерами решения или

Система координат в пространстве - определение с примерами решения.

Эти равенства называют формулами параллельного переноса.

Пример:

В какую точку перейдёт точка Р (-2; 4; 6) при параллельном переносе на векторСистема координат в пространстве - определение с примерами решения = (3; 2; 5)?

Решение:

По вышеприведённым формулам параллельного переноса: Система координат в пространстве - определение с примерами решения.

Ответ: Система координат в пространстве - определение с примерами решения.

Центральная симметрия в пространстве

Если в пространстве Система координат в пространстве - определение с примерами решения, то есть точка О — середина отрезка АА1 то точки А и А1 называют симметричными относительно точки О.

Если в пространстве каждая точка фигуры F переходит в точку, симметричную относительно точки О (рис. 47), то такое преобразование называют симметрией относительно точки О. На рисунках 48, 49 изображёны фигуры симметричные относительно точки О. Симметрия относительно точки является движением.

Если при симметрии относительно точки О фигура F переходит в себя, то её называют центрально симметричной фигурой.

Система координат в пространстве - определение с примерами решения

Например, диагонали параллелепипеда (рис. 50) относительно их точки пересечения О являются центрально симметричными фигурами.

Система координат в пространстве - определение с примерами решения

Пример:

В какую точку перейдет точка A = (1; 2; 3) при симметрии относительно точки О (2; 4; 6)?

Решение:

Пусть А1 = (х; у; z) — искомая точка. По определению точка

О — середина отрезка АА1. Следовательно,

Система координат в пространстве - определение с примерами решения

Из этих уравнений получаем:

Система координат в пространстве - определение с примерами решения.

Ответ: Система координат в пространстве - определение с примерами решения

Симметрия относительно плоскости

Точки А и А1 называют симметричными относительно плоскости а,

если плоскость перпендикулярна отрезку и делит его пополам (рис. 51). Фигуры F1, и F2 на рисунке 52 симметричны относительно

плоскости а. Очевидно, что наш силуэт и его отражение симметричны относительно плоскости зеркала (рис. 53).

Симметрия относительно плоскости а является движением. Система координат в пространстве - определение с примерами решения

Поэтому при симметрии относительно плоскости а отрезок переходит в равный ему отрезок, прямая — в прямую, плоскость — в плоскость.

Если при симмeтрии относительно плоскости фигура F переходит в себя, то её называют фигурой симметричной относительно плоскости.

Например, изображённый на рисунке 54 куб, есть фигура, симметричная относительно плоскости а, проходящей через его диагонали АА1 и СС1.

Поворот и симметрия относительно оси

Система координат в пространстве - определение с примерами решения

Система координат в пространстве - определение с примерами решения

Пусть в пространстве заданы точки А и А1 и прямая l. Если перпендикуляры АК и А1К, опущенные на прямую l, равны и образуют угол Система координат в пространстве - определение с примерами решения, то говорят, что точка А перешла в точку А1 в результате поворота на угол Система координат в пространстве - определение с примерами решения относительно прямой l (рис. 55).

Если каждую точку фигуры F повернуть на угол Система координат в пространстве - определение с примерами решения относительно прямой l, то получим новую фигуру F1 . Тогда говорят, что фигура F перешла в фигуру F1 с помощью поворота на угол Система координат в пространстве - определение с примерами решения относительно прямой l. На рисунке 56 мы видим фигуры, полученные таким поворотом. Например, повернув куб, изображённый на рисунке 57, на 180° относительно прямой l, получим новый куб.

Поворот относительно прямой также является движением.

Поворот на 180° относительно прямой l называют симметрией относительно прямой l.

Центр, ось и плоскость симметрии называют элементами симметрии. Точки, симметричные точке А (х; у; z) относительно координатных плоскостей, координатных осей и начала координат, будут иметь следующие координаты:

Система координат в пространстве - определение с примерами решения

Симметрия в природе и технике

Система координат в пространстве - определение с примерами решения

В природе на каждом шагу можно встретить симметрию.

Например, множество живых существ, в частности тела человека и животных, листья растений и цветы устроены симметрично (рис. 58). Также в неживой природе есть элементы, например, снежинки, кристаллы соли. Молекулярное строение веществ тоже состоит из симметричных фигур. Это, конечно, неспроста, поскольку симметричные фигуры не только красивы, но и самые устойчивые.

Раз так, то можно считать, что красота и совершенство природы построены на основе симметрии. Взяв за основу природную красоту и совершенство, строители, инженеры и архитекторы создают строения и механизмы, здания и сооружения, технику и транспортные средства симметричными. В этой работе им очень помогает наука геометрия.

Подобие пространственных фигур

Пусть Система координат в пространстве - определение с примерами решения и преобразование переводят фигуру F1, в фигуру F2. Если

при этом преобразовании для произвольных точек X1 и Х2 фигуры F1 и соответствующих им точек Y1 и Y2 фигуры Система координат в пространстве - определение с примерами решения, то это преобразование называют преобразованием подобия (рис. 59).

Система координат в пространстве - определение с примерами решения

Как видим, понятие преобразования подобия в пространстве вводится также как на плоскости. Следовательно, рассматриваемые ниже виды подобия, их свойства и доказательства этих свойств подобны соответствующим на плоскости. Поэтому, мы не будем останавливаться на их доказательствах и рекомендуем провести их самостоятельно. Преобразование подобия в пространстве отображает прямую в прямую, луч в луч, отрезок в отрезок и угол в угол. Точно также это преобразование плоскость отображает в плоскость.

Если в пространстве одна из фигур перешла в другую с помощью преобразования подобия, то эти фигуры называют подобными.

Пусть в пространстве задана фигура F, точка О и число к Система координат в пространстве - определение с примерами решения. Преобразование, переводящее произвольную точку X фигуры F в точку Х1 удовлетворяющую условию Система координат в пространстве - определение с примерами решения, называют гомотетией относительно центра О с коэффициентом Система координат в пространстве - определение с примерами решения(рис. 61). Точку О называют центром гомотетии, а число Система координат в пространстве - определение с примерами решения коэффициентом гомотетии. Если в результате такого преобразования каждой точки фигуры F получена фигура F1 то говорят, что фигура F гомотетична фигуре F1.

Вы видите, что определение гомотетии в пространстве аналогично соответствующему определению на плоскости. Следовательно, все свойства и их доказательства аналогичны. Поэтому, мы не будем доказывать их и рекомендуем провести их самостоятельно.

Система координат в пространстве - определение с примерами решения

Гомотетия относительно точки О с коэффициентом Система координат в пространстве - определение с примерами решения является преобразованием подобия. Гомотетия с отличным от нуля коэффициентом Система координат в пространстве - определение с примерами решения при Система координат в пространстве - определение с примерами решения= 1 отображает фигуру F в себя, а при Система координат в пространстве - определение с примерами решения=-1 в фигуру F1 симметричную фигуре F относительно точки О. В остальных случаях гомотетии не сохраняет расстояния между точками, т. е. не является движением. В результате гомотетии расстояние между точками увеличивается в одно и тоже число Система координат в пространстве - определение с примерами решения раз, т. е. меняются измерения фигуры, но сохраняется её форма. При гомотетии а) прямая отображается в параллельную ей прямую (рис. 62.а); b) плоскость — в параллельную ей плоскость (рис. 62.b), если они не проходят через центр гомотетии.

Если же прямая или плоскость проходят через центр гомотетии, то они отображаются в себя.

  • Иррациональные числа
  • Действительные числа
  • Решение уравнений высших степеней
  • Системы неравенств
  • Уравнения и неравенства
  • Уравнения и неравенства содержащие знак модуля
  • Уравнение
  • Метод математической индукции

Прямоугольная система координат в пространстве.

Прямоугольная система координат в пространстве.

План

1. Прямоугольная система координат в пространстве.

2. Расстояние между точками

3. Координаты середины отрезка

Вопрос 1. Прямоугольная система координат в пространстве.

Если через точку О в пространстве мы проведем три перпендикулярные прямые, назовем их, выберем направление, обозначим единичные отрезки, то мы получим прямоугольную систему координат в пространстве. Оси координат называются так: Ох – ось абсцисс, Оy – ось ординат и Оz – ось аппликат. Вся система координат обозначается – Oxyz. Таким образом, появляются три координатные плоскости: Оxy, Оxz, Оyz.

Пример построения точки В(4;3;5) в прямоугольной системе координат:

Первая координата точки B – 4, поэтому откладываем на Ox 4, проводим прямую параллельно оси Oy до пересечения с прямой, проходящей через у=3. Таким образом, мы получаем точку K. Эта точка лежит в плоскости Oxy и имеет координаты K(4;3;0). Теперь нужно провести прямую параллельно оси Oz. И прямую, которая проходит через точку с аппликатой 5 и параллельна диагонали параллелограмма в плоскости Oxy. На их пересечении мы получим искомую точку B.

Рассмотрим расположение точек, у которых одна или две координаты равны.

Например, точка A(3;-1;0). Нужно продолжить ось Oy влево до значения -1, найти точку 3 на оси Ox, и на пересечении линий, проходящих через эти значения, получаем точку А. Эта точка имеет аппликату 0, а значит, она лежит в плоскости Oxy.

Точка C(0;2;0) имеет абсциссу и аппликату 0 – не отмечаем. Ордината равна 2, значит точка C лежит только на оси Oy, которая является пересечением плоскостей Oxy и Oyz.

Чтобы отложить точку D(-4;0;3) продолжаем ось Ox назад за начало координат до точки -4. Теперь восстанавливаем из этой точки перпендикуляр – прямую, параллельную оси Oz до пересечения с прямой, параллельной оси Ox и проходящей через значение 3 на оси Oz. Получаем точку D(-4;0;3). Так как ордината точки равна 0, значит точка D лежит в плоскости Oxz.

Следующая точка E(0;5;-3). Ордината точки 5, аппликата -3, проводим прямые проходящие через эти значения на соответствующих осях, и на их пересечении получаем точку E(0;5;-3). Эта точка имеет первую координату 0, значит она лежит в плоскости Oyz.

Вопрос 2. Расстояние между точками

Выразим расстояние между двумя точками  и  через координаты этих точек.

Рассмотрим сначала случай, когда прямая  не параллельна оси z.

Проведём через точки  и  прямые, параллельные оси z. Они пересекут плоскость в точках  и . Эти точки имеют те же координаты x, y, что и точки  и , а координата z у них равна нулю. Проведём теперь плоскость через точку , параллельную оси xy. Она пересечёт прямую  в некоторой точке C. По теореме Пифагора . Отрезки  и  равны, а . Длина отрезка  равна . Поэтому . Если отрезок  не параллелен оси z, то . Тот же результат даёт полученная формула, так как в этом случае .

Расстояние между двумя точками вычисляется по формуле

Вопрос 3.Координаты середины отрезка

Середина отрезка — это точка, которая лежит на отрезке и находится на равном расстоянии от конечных точек.

В геометрических задачах часто можно столкнуться с необходимостью найти середину отрезка заданного координатами точек его концов, например в задачах о поиске медианы или средней линии.

Каждая координата середины отрезка равна полусумме соответствующих координат концов отрезка.

То есть координаты середины отрезка с концами  и  вычисляются по формулам:

Вы уже знакомы с прямоугольной системой координат на
плоскости
, другими словами прямоугольной координатной плоскостью. Такую
систему координат задают две взаимно перпендикулярные прямые, на каждой
из которых выбрано направление и величина единичного отрезка. Эти
прямые называют осями абсцисс и ординат.

Точку пересечения осей называют точкой начала координат.

Прямоугольную систему координат на плоскости
обозначают Оху.

Каждой точке плоскости сопоставляется только одна
пара чисел, которые называют её координатами. Для определения координат,
из точки нужно провести перпендикуляры к осям, тем самым мы и получим абсциссу
и ординату точки.

Определение:

Если же через точку пространства проведены
три попарно перпендикулярные прямые, а на каждой из них выбрано направление и
единичный отрезок, то говорят, что задана прямоугольная система координат
в пространстве
.

Прямые с выбранными на них направлениями называют осями
координат
, а точку их пересечения — началом координат.
Как и на плоскости её обычно обозначают буквой О.

Оси координат обозначают так: Ох, Оу, Оz.
И называют осью абсцисс, осью ординат и, новым
является название третьей оси, ось аппликат.

Прямоугольную систему координат в пространстве
обозначают Охуz.

Через каждые 2 оси координат проходят координатные
плоскости: Оху, Оуz и Охz.
Всего таких плоскостей 3.

Каждая ось делится точкой О на два луча. В
соответствии с этим, лучи, направление которых совпадает с направлением оси,
называют положительными полуосями, а оставшиеся лучи — отрицательными
полуосями
.

Каждой точке пространства сопоставляется только одна
тройка чисел, которые называют её координатами. Их определяют
аналогично тому, как это делали на плоскости. Только через точку М проводят
плоскости перпендикулярные координатным осям.

Точки пересечения проведённых плоскостей с осями
координат назовём М1, М2 и М3.

Первая координата точки М, то есть её абсцисса,
равна длине отрезка ОМ1.

Вторая координата, которую называют ординатой, равна
длине отрезка ОМ2.

Ну, а третья координата, а точнее аппликата, равна
длине отрезка ОМ3.

Координаты точки записывают в скобках, при этом
первой записывают абсциссу, второй — ординату, а третьей — аппликату.

В данном случае точки М1, М2 и
М3 являются точками положительных полуосей, поэтому и координаты
точки М будут положительными числами.

Рассмотрим примеры различного расположения точек в
прямоугольной системе координат.

Задание:
определить координаты точек А, В, С, D,
Е и F.

После выполнения этого задания можно сделать вывод
о том, что если точка лежит в некоторой координатной плоскости или на
некоторой координатной оси, то её соответствующие координаты будут равны нулю
.

Так если точка лежит в координатной плоскости
ОИксИгрек, то её аппликата равна нулю. Если точка лежит в координатной
плоскости ОИксЗэт, то её ордината равна нулю. И если точка лежит в координатной
плоскости ОИгрекЗэт, то её абсцисса равна нулю.

Ну, а в случаях, когда точка лежит на одной из осей,
только одна координата является ненулевой.

Задание:
По координатам точек 𝐴(3;−1;0), 𝐵(0;0;−7),
𝐶(2;0;0),
𝐷(−4;0;3),
𝐸(0;−1;0),
𝐹(1;2;3),
𝐺(0;5−7),
𝐻(−√5;√3;0)
определить, какие из них лежат на той или иной координатной оси или в той или
иной координатной плоскости.

Решение:

Задание:
найти координаты проекций точки 𝐴(2;−3;5)
на каждую из координатных плоскостей и на каждую из координатных осей.

Далее найдём координаты проекций точки А на
координатные плоскости.

Проекцией точки А на координатную плоскость Оху
является основание перпендикуляра, проведённого из точки А к данной
координатной плоскости. При этом координаты полученной проекции будут такими же
как у точки А, только аппликата станет равной нулю.

Аналогично получим проекцию точки А на координатную
плоскость Оуz. Проведём
перпендикуляр из данной точки к данной координатной плоскости. Его основание и
является проекцией точки А на плоскость Оуz.
Координаты данной проекции равны координатам точки А, только абсцисса равна
нулю.

Ну, а проекция точки А на координатную плоскость Охz
будет иметь координаты 2, 0, 5.

Так мы с вами нашли координаты проекций точки А на
координатные оси и на координатные плоскости.

Задание:
𝐴𝐵𝐶𝐷𝐴1𝐵1𝐶1𝐷1
куб; 𝐴(0;0;0),
𝐵(0;0;1),
𝐷(0;1;0),
𝐴1
(1;0;0). Найти координаты точек 𝐶,
𝐵1,
𝐶1
и 𝐷1.

Решение:
Изобразим прямоугольную систему координат. Отметим точки, являющиеся вершинами
куба, координаты которых известны.

Итоги:

На этом уроке вы познакомились с понятием
прямоугольной системы координат в пространстве. Узнали, что её задают три
взаимно перпендикулярные прямые, на которых выбраны направления и единичные
отрезки. Эти прямые называют координатными осями. Точку пересечения осей
называют точкой начала координат.

Ось Ох называют осью абсцисс, ось Оу называют осью
ординат, и новым для вас является название оси Оz
— ось аппликат. Помимо осей координат в прямоугольной системе координат
присутствуют и координатные плоскости: Оху, Оуz
и Охz.

Всю прямоугольную систему координат в пространстве
обозначают Охуz.

Любой точке пространства соответствует только одна
тройка чисел х, у и z, которые и
являются её координатами. Все координаты точки О начала координат равны нулю.

Система координат

С чего было бы логично начать обсуждение метода координат? Наверное, с понятия системы координат. Вспомни, когда ты с нею впервые столкнулся.

Мне кажется, что в 7 классе, когда ты узнал про существование линейной функции ( y=ax+b), например, ( y=2{x}-3).

Напомню, ты строил ее по точкам. Помнишь?

Ты выбирал произвольное число ( x), подставлял ее в формулу ( y=2{x}-3) и вычислял таким образом ( y).

Например, если ( x=0), то ( y=2cdot 0-3=-3), если же ( x=1), то ( y=2cdot 1-3=-1)и т. д.

Что же ты получал в итоге?

А получал ты точки с координатами: ( Aleft( 0,-3 right)) и ( Bleft( 1,-1 right)).

Далее ты рисовал «крестик» (систему координат ( X0Y)), выбирал на ней масштаб (сколько клеточек у тебя будет единичным отрезком) и отмечал на ней полученные тобою точки, которые затем соединял прямой линией, полученная линия и есть график функции ( y=2{x}-3).

Тут есть несколько моментов, которые стоит объяснить тебе чуть подробнее:

  • Единичный отрезок ты выбираешь из соображений удобства, так, чтобы все красиво и компактно умещалось на рисунке;
  • Принято, что ось ( displaystyle X) идет слева направо, а ось ( displaystyle Y) – cнизу вверх;
  • Они пересекаются под прямым углом, а точка их пересечения называется началом координат. Она обозначается буквой ( displaystyle O);
  • В записи координаты точки, например ( displaystyle Aleft( 0,-3 right)), слева в скобках стоит координата точки по оси ( displaystyle X), а справа, по оси ( displaystyle Y). В частности, ( displaystyle Aleft( 0,-3 right)) просто означает, что у точки ( displaystyle A) ( displaystyle x=0,~y=-3.);
  • Для того, чтобы задать любую точку на координатной оси, требуется указать ее координаты (2 числа);
  • Для любой точки, лежащей на оси ( displaystyle Ox,), ( displaystyle y=0.);
  • Для любой точки, лежащей на оси ( displaystyle Oy), ( displaystyle x=0.);
  • Ось ( displaystyle Ox) называется осью абсцисс;
  • Ось ( displaystyle Oy) называется осью ординат.

Векторы

Теперь давай с тобой сделаем следующий шаг: отметим две точки ( displaystyle Aleft( {{x}_{1}},{{y}_{1}} right)) ( displaystyle Bleft( {{x}_{2}},{{y}_{2}} right)).

Соединим эти две точки отрезком. И поставим стрелочку так, как будто мы проводим отрезок из точки ( displaystyle A) к точке ( displaystyle B):

То есть мы сделаем наш отрезок направленным!

Вспомни, как еще называется направленный отрезок? Верно, он называется вектором!

Вектором называется направленный отрезок, имеющий начало и конец.

Таким образом, если мы соединим точку ( displaystyle A) c точкой ( displaystyle B), причем началом у нас будет точка A, а концом – точка B, то мы получим вектор ( displaystyle overrightarrow{AB}).

Это построение ты тоже делал в 8 классе, помнишь?

Координаты вектора

Оказывается, векторы, как и точки, можно обозначать двумя цифрами: эти цифры называются координатами вектора.

Вопрос: как ты думаешь, достаточно ли нам знать координаты начала и конца вектора, чтобы найти его координаты?

Оказывается, что да! И делается это очень просто:

Координаты вектора = координаты точки конца – координаты точки начала.

Таким образом, так как в векторе ( displaystyle overrightarrow{AB}) точка ( displaystyle Aleft( {{x}_{1}},{{y}_{1}} right)) – начало, а ( displaystyle Bleft( {{x}_{2}},{{y}_{2}} right)) – конец, то вектор ( displaystyle overrightarrow{AB}) имеет следующие координаты:

( displaystyle overrightarrow{AB}left( {{x}_{2}}-{{x}_{1}},{{y}_{2}}-{{y}_{1}} right))

Например, если ( displaystyle Aleft( 2,0 right))( displaystyle Bleft( 1,2 right)), то координаты вектора ( displaystyle overrightarrow{AB})

( displaystyle overrightarrow{AB}left( 1-2,2-0 right)=overrightarrow{AB}left( -1,2 right))

Теперь давай сделаем наоборот, найдем координаты вектора ( displaystyle overrightarrow{BA}).

Что нам для этого нужно поменять? Да, нужно поменять местами начало и конец: теперь начало вектора будет в точке ( displaystyle B), а конец – в точке ( displaystyle A).

Тогда:

( displaystyle overrightarrow{BA}left( 2-1,text{ }!!~!!text{ }0-2 right)=overrightarrow{BA(}1,-2).)

Посмотри внимательно, чем отличаются векторы ( displaystyle overrightarrow{AB}) и ( displaystyle overrightarrow{BA})?

Единственное их отличие – это знаки в координатах. Они противоположны. Этот факт принято записывать вот так:

( displaystyle overrightarrow{AB}=-overrightarrow{BA})

Иногда, если не оговаривается специально, какая точка является началом вектора, а какая – концом, то векторы обозначают не двумя заглавными буквами, а одной строчной, например: ( displaystyle {vec{a}}), ( displaystyle {vec{p}}) и т. д.

Еще больше о векторах и проекциях (эту тему мы непременно затронем) ты можешь прочитать в статье по физике «Большая теория по векторам» 🙂

Действия с векторами

Что еще можно делать с векторами?

Да почти все то же самое, что и с обычными числами:

  • Векторы можно складывать друг с другом;
  • Векторы можно вычитать друг из друга;
  • Векторы можно умножать (или делить) на произвольное ненулевое число;
  • Векторы можно умножать друг на друга.

Что же происходит при выполнении этих действий с координатами векторов?

1. При сложении (вычитании) двух векторов, мы складываем (вычитаем) поэлементно их координаты.

То есть:

( vec{a}left( {{x}_{1}},{{y}_{1}} right)+vec{b}left( {{x}_{2}},{{y}_{2}} right)=vec{c}left( {{x}_{1}}+{{x}_{2}},{{y}_{1}}+{{y}_{2}} right))

( vec{a}left( {{x}_{1}},{{y}_{1}} right)-vec{b}left( {{x}_{2}},{{y}_{2}} right)=vec{c}left( {{x}_{1}}-{{x}_{2}},{{y}_{1}}-{{y}_{2}} right))

2. При умножении (делении) вектора на число, все его координаты умножаются (делятся) на это число:

( kcdot vec{a}left( {{x}_{1}},{{y}_{1}} right)=vec{b}left( k{{x}_{1}},k{{y}_{1}} right))

Например:

Най­ди­те сумму ко­ор­ди­нат век­то­ра ( vec{a}+vec{b}).

Вектор растягивается или сжимается или меняет направление при умножении или делении на число:

Давай вначале найдем координаты каждого из векторов.

Оба они имеют одинаковое начало – точку начала координат. Концы у них разные.

Тогда ( vec{a}left( 2-0,6-0 right)=vec{a}left( 2,6 right)), ( vec{b}left( 8-0,4-0 right)=vec{b}left( 8,4 right)).

Теперь вычислим координаты вектора ( vec{c}=vec{a}+vec{b}=vec{c}left( 2+8,4+6 right)=vec{c}left( 10,10 right))

Тогда сумма координат полученного вектора равна ( 20).

Ответ: ( 20)

Теперь реши сам следующую задачу:

Найти сумму координат вектора ( 3vec{a}-2vec{b})

Проверяем:

  • ( vec{a}=vec{a}left( 4-2,10-4 right)=vec{a}left( 2,6 right));
  • ( vec{b}=vec{b}left( 10-2,6-2 right)=vec{b}left( 8,4 right));
  •  ( vec{c}=3vec{a}-2vec{b}=3vec{a}left( 2,6 right)-2vec{b}left( 8,4 right)=left( 6,18 right)-left( 16,8 right)=vec{c}left( -10,10 right)); 
  • ( -10+10=0).

Ответ: ( 0)

Расстояние между двумя точками на координатной плоскости

Давай рассмотрим теперь следующую задачу: у нас есть две точки на координатной плоскости. Как найти расстояние между ними?

Пусть первая точка будет ( {{P}_{1}}({{x}_{1}},{{y}_{1}})), а вторая ( {{P}_{2}}left( {{x}_{2}},{{y}_{2}} right)).

Обозначим расстояние между ними через ( d). Давай сделаем для наглядности следующий чертеж:

Что я сделал?

Я, во-первых, соединил точки ( {{P}_{1}}left( {{x}_{1}},{{y}_{1}} right)) и ( {{P}_{2}}left( {{x}_{2}},{{y}_{2}} right)).

А также из точки ( {{P}_{1}}) провел линию, параллельную оси ( Ox), а из точки ( {{P}_{2}}) провел линию, параллельную оси ( Oy).

Они пересеклись в точке ( R), образовав при этом замечательную фигуру. Чем она замечательна?

Да мы с тобой почти что все знаем про прямоугольный треугольник. Ну уж теорему Пифагора – точно!

Искомый отрезок – это гипотенуза этого треугольника, а отрезки ( {{P}_{1}}R,~{{P}_{2}}R) – катеты.

Чему равны координаты точки ( R)?

Да, их несложно найти по картинке: ( Rleft( {{x}_{2}},{{y}_{1}} right).~)

Так как отрезки ( {{P}_{1}}R,~{{P}_{2}}R) параллельны осям ( Ox) и ( Oy) соответственно, то их длины легко найти: если обозначить длины отрезков ( {{P}_{1}}R,~{{P}_{2}}R) соответственно через ( left| {{P}_{1}}Rleft| ,~ right|{{P}_{2}}R right|), то

( left| {{P}_{1}}R right|={{x}_{2}}-{{x}_{1}})

( left| {{P}_{2}}R right|={{y}_{2}}-{{y}_{1}})

Теперь воспользуемся теоремой Пифагора. Длины катетов нам известны, гипотенузу мы найдем:

( {{d}^{2}}=text{ }!!~!!text{ }left| {{P}_{1}}{{P}_{2}} right|=text{ }!!~!!text{ }{{left| {{P}_{1}}R right|}^{2}}+{{left| {{P}_{2}}R right|}^{2}}=({{x}_{2}}-{{x}_{1}}){{~}^{2}}+({{y}_{2}}-{{y}_{1}}){{~}^{2}}~)

( d=~sqrt{({{x}_{2}}-{{x}_{1}}){{~}^{2}}+({{y}_{2}}-{{y}_{1}}){{~}^{2}}})

Таким образом, расстояние между двумя точками – это корень из суммы квадратов разностей из координат. 

Или же – расстояние между двумя точками – это длина отрезка, их соединяющего.

Легко заметить, что расстояние между точками не зависит от направления.

Тогда:

( d=left| overrightarrow{{{P}_{1}}{{P}_{2}}} right|=left| overrightarrow{{{P}_{2}}{{P}_{1}}} right|=sqrt{({{x}_{2}}-{{x}_{1}}){{~}^{2}}+({{y}_{2}}-{{y}_{1}}){{~}^{2}}})

Отсюда делаем три вывода:

  • Длина вектора = корень из суммы квадратов его координат;
  • Найти расстояние между двумя точками = найти длину вектора, их соединяющего (в любом направлении);
  • Длины векторов, соединяющих две точки в разном направлении, равны.

Давай немного поупражняемся в вычислении расстояния между двумя точками:

Например, если ( Aleft( 1,2 right),~Bleft( 3,4 right)), то расстояние между ( A) и ( B) равно

( d=sqrt{{{left( 3-1 right)}^{2}}+{{left( 4-2 right)}^{2}}}=sqrt{4+4}=sqrt{8}=2sqrt{2})

Или пойдем по-другому: найдем координаты вектора ( overrightarrow{AB})

( overrightarrow{AB}left( 3-1,4-2 right)=overrightarrow{AB}left( 2,2 right))

И найдем длину вектора:

( left| overrightarrow{AB} right|=sqrt{{{2}^{2}}+{{2}^{2}}}=sqrt{8}=2sqrt{2})

Как видишь, одно и то же!

Теперь немного потренируйся сам:

Задание. Найти расстояние между указанными точками:

  • ( Aleft( 2,sqrt{3} right),~Bleft( 5,2sqrt{3} right));
  • ( Cleft( 2,4 right),~Dleft( 1,-5 right));
  • ( Fleft( sqrt{12},1 right),~Gleft( sqrt{3},-1 right)).

Проверяем:

  • ( d=sqrt{{{left( 5-2 right)}^{2}}+{{left( 2sqrt{3}-sqrt{3} right)}^{2}}}=sqrt{9+3}=sqrt{12}=2sqrt{3});
  • ( displaystyle d=sqrt{{{left( 1-2 right)}^{2}}+{{left( -5-4 right)}^{2}}}=sqrt{1+81}=sqrt{82});
  • ( displaystyle d=sqrt{{{left( sqrt{3}-sqrt{12} right)}^{2}}+{{left( -1-1 right)}^{2}}}=sqrt{left( 3-2sqrt{3}sqrt{12}+12 right)+4}=); ( displaystyle=sqrt{3-2sqrt{36}+12+4}=sqrt{3-12+12+4}=sqrt{7}).

Вот еще пара задачек на ту же формулу, правда звучат они немного по-другому:

1. Най­ди­те квад­рат длины век­то­ра ( vec{a}-vec{b}).

2. Най­ди­те квад­рат длины век­то­ра ( overrightarrow{AB})

Я так думаю, ты с ними без труда справился? Проверяем:

1. А это на внимательность) Мы уже нашли координаты векторов ( displaystyle {vec{a}}) и ( displaystyle {vec{b}}) ранее: ( displaystyle vec{a}left( 2,6 right),~vec{b}left( 8,4 right)). Тогда вектор ( displaystyle vec{a}-vec{b}) имеет координаты ( displaystyle left( 2-8,6-4 right)=left( -6,2 right)). Квадрат его длины будет равен:

( displaystyle {{d}^{2}}={{left( -6 right)}^{2}}+{{2}^{2}}=36+4=40.)

2. Найдем координаты вектора ( displaystyle overrightarrow{AB}=overrightarrow{AB}left( 8-2,6-4 right)=overrightarrow{AB}left( 6,2 right))

Тогда квадрат его длины равен

( displaystyle {{d}^{2}}={{6}^{2}}+{{2}^{2}}=36+4=40.)

Ничего сложного, правда? Обычная арифметика, не более того.

Следующие задачки нельзя однозначно классифицировать, они скорее на общую эрудицию и на умение рисовать простенькие картинки.

Задача 1. Най­ди­те синус угла на­кло­на от­рез­ка, со­еди­ня­ю­ще­го точки ( displaystyle Oleft( 0;~0 right)),( displaystyle Aleft( 6;~8 right)) с осью абсцисс.

Как мы будем поступать здесь?

Нужно найти синус угла между ( displaystyle OA) и осью ( displaystyle Ox).

А где мы умеем искать синус? Верно, в прямоугольном треугольнике.

Так что нам нужно сделать? Построить этот треугольник!

Поскольку координаты точки ( displaystyle A-6) и ( displaystyle 8), то отрезок ( displaystyle OB) равен ( displaystyle 6), а отрезок ( displaystyle AB-8).

Нам нужно найти синус угла ( displaystyle angle AOB).

Напомню тебе, что синус – это отношение противолежащего катета к гипотенузе, тогда

( displaystyle sinangle AOB=frac{AB}{OA})

Что нам осталось сделать?

Найти гипотенузу.

Ты можешь сделать это двумя способами: по теореме Пифагора (катеты-то известны!) или по формуле расстояния между двумя точками (на самом деле одно и то же, что и первый способ!).

Я пойду вторым путем:

( displaystyle OA=sqrt{{{left( 6-0 right)}^{2}}+{{left( 8-0 right)}^{2}}}=10)

Тогда

( displaystyle sinangle AOB=frac{AB}{OA}=frac{8}{10}=0.8)

Ответ: ( displaystyle 0.8)

Следующая задача покажется тебе еще проще. Она – на координаты точки.

Задача 3. В условиях предыдущей задачи найти сумму расстояний от точки ( displaystyle A) до осей координат.

Задача – вообще элементарная, если знать, что такое расстояние от точки до осей.

Ты знаешь?

Я надеюсь, но все же напомню тебе:

Расстояние от точки до осей координат – это длины перпендикуляров, опущенных из точки к осям.

Итак, на моем рисунке, расположенном чуть выше, я уже изобразил один такой перпендикуляр. К какой он оси?

К оси ( displaystyle Ox).

И чему же равна тогда его длина?

Она равна ( displaystyle 8).

Теперь сам проведи перпендикуляр к оси ( displaystyle Oy) и найди его длину. Она будет равна ( displaystyle 6), ведь так?

Тогда их сумма равна ( displaystyle 14).

Ответ: ( displaystyle 14).

Задача 4. В условиях задачи 2, найдите ординату точки, симметричной точке ( displaystyle A) относительно оси абсцисс.

Решение:

Я думаю, тебе интуитивно ясно, что такое симметрия?

Очень многие объекты ею обладают: многие здания, столы, самолеты, многие геометрические фигуры: шар, цилиндр, квадрат, ромб и т. д.

Грубо говоря, симметрию можно понимать вот как: фигура состоит из двух (или более) одинаковых половинок. Такая симметрия называется осевой.

А что тогда такое ось?

Это как раз та линия, по которой фигуру можно, условно говоря, «разрезать» на одинаковые половинки (на данной картинке ось симметрии – прямая ( displaystyle l)):

Теперь давай вернемся к нашей задаче.

Нам известно, что мы ищем точку, симметричную относительно оси ( displaystyle Ox).

Тогда эта ось – ось симметрии.

Значит, нам нужно отметить такую точку ( displaystyle {{A}_{1}}), чтобы ось ( displaystyle Ox) разрезала отрезок ( displaystyle A{{A}_{1}}) на две равные части.

Попробуй сам отметить такую точку. А теперь сравни с моим решением:

У тебя получилось так же?

Хорошо! У найденной точки нас интересует ордината.

Она равна ( displaystyle -8)

Ответ: ( displaystyle -8)

Теперь задачка на параллелограмм:

Задача 5. Точки ( displaystyle Oleft( 0;~0 right),~Aleft( 6;~8 right),~Cleft( 0;~6 right)~) яв­ля­ют­ся вер­ши­на­ми па­рал­ле­ло­грам­ма. Най­ди­те ор­ди­на­ту точки ( displaystyle B).

Можно решать эту задачу двумя способами: логикой и методом координат. 

Я вначале применю метод координат, а потом расскажу тебе, как можно решить иначе.

Совершенно ясно, что абсцисса точки ( displaystyle B) равна ( displaystyle 6). (она лежит на перпендикуляре, проведенной из точки ( displaystyle A) к оси абсцисс).

Нам нужно найти ординату.

Воспользуемся тем, что наша фигура – параллелограмм, это значит, что ( displaystyle CA=OB).

Найдем длину отрезка ( displaystyle CA), используя формулу расстояния между двумя точками:

( d=sqrt{{{left( 6-0 right)}^{2}}+{{left( 8-6 right)}^{2}}}=sqrt{40})

Тогда ( OB=sqrt{40}.~~)

Опускаем перпендикуляр, соединяющий точку ( B) с осью ( Ox).

Точку пересечения обозначу буквой ( D).

Длина отрезка ( OD) равна ( 6). (найди сам задачу, где мы обсуждали этот момент), тогда найдем длину отрезка ( BD) по теореме Пифагора:

( BD=sqrt{40-36}=2)

Длина отрезка – в точности совпадает с его ординатой.

Ответ: ( 2).

Другое решение (я просто приведу рисунок, который его иллюстрирует)

Ход решения:

  • Провести ( CE);
  • Найти координаты точки ( E) и длину ( AE);
  • Доказать, что ( BD=AE).

Еще одна задачка на длину отрезка:

Точки ( Oleft( 0;~0 right),~Aleft( 6;~8 right),~Bleft( 8;~2 right)) яв­ля­ют­ся вер­ши­на­ми тре­уголь­ни­ка. Най­ди­те длину его сред­ней линии ( CD), па­рал­лель­ной ( OA).

Ты помнишь, что такое средняя линия треугольника?

Тогда для тебя эта задача элементарна. Если не помнишь, то я напомню: средняя линия треугольника – это линия, которая соединяет середины противоположных сторон.

Она параллельна основанию и равна его половине.

Основание – это отрезок ( OA).

Его длину нам приходилось искать ранее, оно равно ( 10).

Тогда длина средней линии вдвое меньше и равна ( 5).

Ответ: ( 5).

Комментарий: эту задачу можно решить и другим способом, к которому мы обратимся чуть позже.

А пока – вот тебе несколько задачек, потренируйся на них, они совсем простые, но помогают «набивать руку», на использовании метода координат!

1. Точки ( Oleft( 0;~0 right),~Aleft( 10;~0 right),~Bleft( 8;~6 right),~Cleft( 2;~6 right)) яв­ля­ют­ся вер­ши­на­ми тра­пе­ции. Най­ди­те длину ее сред­ней линии ( DE).

2. Точки ( Oleft( 0;~0 right),~Bleft( 8;~2 right),~Cleft( 2;~6 right)) и ( A) яв­ля­ют­ся вер­ши­на­ми па­рал­ле­ло­грам­ма. Най­ди­те ор­ди­на­ту точки ( A).

3. Най­ди­те длину от­рез­ка, со­еди­ня­ю­ще­го точки ( Aleft( 6 ;~8 right)) и ( Bleft( -2;~2 right).)

4. Най­ди­те пло­щадь за­кра­шен­ной фи­гу­ры на ко­ор­ди­нат­ной плос­ко­сти.

5. Окруж­ность с цен­тром в на­ча­ле ко­ор­ди­нат про­хо­дит через точку ( displaystyle Pleft( 8;text{ }6 right)). Най­ди­те ее ра­ди­ус.

6. Най­ди­те ра­ди­ус окруж­но­сти, опи­сан­ной около пря­мо­уголь­ни­ка ( displaystyle ABCD), вер­ши­ны ко­то­ро­го имеют ко­ор­ди­на­ты со­от­вет­ствен­но ( displaystyle left( -2;~-2 right),~left( 6;~-2 right),~left( 6;~4 right),~left( -2;~4 right).)

Решения:

1. Известно, что средняя линия трапеции равна полусумме ее оснований.

Основание ( displaystyle CB) равно ( displaystyle 6), а основание ( displaystyle OA-10).

Тогда ( displaystyle ED=frac{CB+OA}{2}=frac{16}{2}=8)

Ответ: ( displaystyle 8)

2. Проще всего решить эту задачу так: заметить, что ( displaystyle overrightarrow{OA}=overrightarrow{OC}+overrightarrow{OB}) (правило параллелограмма).

Вычислить координаты векторов ( displaystyle overrightarrow{OC}) и ( displaystyle overrightarrow{OB}) не представляет труда: ( displaystyle overrightarrow{OC}left( 2,6 right),~overrightarrow{OB}left( 8,2 right)).

При сложении векторов координаты складываются.

Тогда ( displaystyle overrightarrow{OA}) имеет координаты ( displaystyle left( 10,8 right)).

Эти же координаты имеет и точка ( displaystyle A), поскольку начало вектора ( displaystyle overrightarrow{OA}) – это точка с координатами ( displaystyle left( 0,0 right)).

Нас интересует ордината. Она равна ( displaystyle 8).

Ответ: ( displaystyle 8)

3. Действуем сразу по формуле расстояния между двумя точками:

( displaystyle d=sqrt{{{left( 6-left( -2 right) right)}^{2}}+{{left( 8-2 right)}^{2}}}=sqrt{64+36}=10)

Ответ: ( displaystyle 10)

4. Посмотри на картинку и скажи, между какими двумя фигурами «зажата» заштрихованная область?

Она зажата между двумя квадратами. Тогда площадь искомой фигуры равна площади большого квадрата минус площадь маленького.

Сторона маленького квадрата – это отрезок, соединяющий точки ( displaystyle left( 0,2 right)) и ( displaystyle left( 2,0 right).) Его длина равна

( displaystyle {{d}_{1}}=sqrt{{{left( 0-2 right)}^{2}}+{{left( 2-0 right)}^{2}}}=sqrt{8})

Тогда площадь маленького квадрата равна

( displaystyle {{S}_{1}}=d_{1}^{2}={{sqrt{8}}^{2}}=8)

Точно так же поступаем и с большим квадратом: его сторона – это отрезок, соединяющий точки ( displaystyle left( 0,4 right)) и ( displaystyle left( 4,0 right).)

Его длина равна

( displaystyle {{d}_{2}}=sqrt{{{left( 0-4 right)}^{2}}+{{left( 4-0 right)}^{2}}}=sqrt{32}).

Тогда площадь большого квадрата равна

( displaystyle {{S}_{2}}=d_{2}^{2}={{sqrt{32}}^{2}}=32)

Площадь искомой фигуры найдем по формуле:

( displaystyle S={{S}_{2}}-{{S}_{1}}=32-8=24)

Ответ: ( displaystyle 24)

5. Если окружность имеет в качестве центра начало координат и проходит через точку ( displaystyle P), то ее радиус ( displaystyle R) будет в точности равен длине отрезка ( displaystyle OP) (сделай рисунок и ты поймешь, почему это очевидно).

Найдем длину этого отрезка:

( displaystyle R=sqrt{{{6}^{2}}+{{8}^{2}}}=10)

Ответ: ( displaystyle 10)

6. Известно, что радиус описанной около прямоугольника окружности равен половине его диагонали.

Найдем длину любой из двух диагоналей (ведь в прямоугольнике они равны!)

( displaystyle left| AC right|=sqrt{{{left( 6-left( -2 right) right)}^{2}}+{{left( 4-left( -2 right) right)}^{2}}}=10)

Тогда

( displaystyle R=frac{1}{2}left| AC right|=5)

Ответ: ( displaystyle 5)

Ну что, ты со всем справился?

Было не очень сложно разобраться, ведь так? Правило здесь одно – уметь сделать наглядную картинку и просто «считать» с нее все данные.

Нам осталось совсем немного. Есть еще буквально два момента, которые бы мне хотелось обсудить:

  • как найти координаты середины отрезка и

Координаты середины отрезка

Давай попробуем решить вот такую нехитрую задачку.

Пусть даны две точки ( displaystyle Aleft( {{x}_{1}},{{x}_{2}} right)~) и ( displaystyle Bleft( {{x}_{2}},{{y}_{2}} right)).

Найти координаты середины отрезка ( displaystyle AB). Решение этой задачки следующее: пусть точка ( displaystyle D) – искомая середина, тогда ( displaystyle D) имеет координаты:

( displaystyle Dleft( frac{{{x}_{1}}+{{x}_{2}}}{2},frac{{{y}_{1}}+{{y}_{2}}}{2} right))

То есть: координаты середины отрезка = среднее арифметическое соответствующих координат концов отрезка.

Это правило очень простое и как правило не вызывает затруднений у учащихся. Давай посмотрим, в каких задачках и как оно употребляется:

1. Най­ди­те ор­ди­на­ту се­ре­ди­ны от­рез­ка, со­еди­ня­ю­ще­го точки ( displaystyle Aleft( 6,~8 right)~) и ( displaystyle Bleft( -2,~2 right).)

2. Точки ( displaystyle Oleft( 0;~0 right),~Aleft( 6;~8 right),~Bleft( 6;~2 right),~Cleft( 0;~6 right)) яв­ля­ют­ся вер­ши­на­ми че­ты­рех­уголь­ни­ка. Най­ди­те ор­ди­на­ту точки ( displaystyle P) пе­ре­се­че­ния его диа­го­на­лей.

3. Най­ди­те абс­цис­су цен­тра окруж­но­сти, опи­сан­ной около пря­мо­уголь­ни­ка ( displaystyle ABCD), вер­ши­ны ко­то­ро­го имеют ко­ор­ди­на­ты со­от­вет­ствен­но ( displaystyle left( -2;~-2 right),~left( 6;~-2 right),~left( 6;~4 right),~left( -2;~4 right)).

Решения:

1. Первая задачка – просто классика. Действуем сразу по определению середины отрезка. Она имеет координаты ( displaystyle left( frac{6-2}{2},~frac{8+2}{2} right)=left( 2,5 right)).

Ордината равна ( displaystyle 5).

Ответ: ( displaystyle 5)

2. Легко видеть, что данный четырехугольник является параллелограммом (даже ромбом!). Ты и сам можешь это доказать, вычислив длины сторон и сравнив их между собой.

Что я знаю про параллелограмм?

Его диагонали точкой пересечения делятся пополам! Ага! Значит точка пересечения диагоналей – это что?

Это середина любой из диагоналей!

Выберу, в частности диагональ ( displaystyle OA). Тогда точка ( displaystyle P) имеет координаты ( displaystyle left( frac{6+0}{2},frac{8+0}{2} right)=left( 3,4 right).)

Ордината точки ( displaystyle P) равна ( displaystyle 4).

Ответ: ( displaystyle 4)

3. С чем совпадает центр описанной около прямоугольника окружности?

Он совпадает с точкой пересечения его диагоналей. А что ты знаешь про диагонали прямоугольника?

Они равны и точкой пересечения делятся пополам. Задача свелась к предыдущей.

Возьму, например, диагональ ( displaystyle AC). Тогда если ( displaystyle P) – центр описанной окружности, то ( displaystyle P) – середина ( displaystyle AC).

Ищу координаты: ( displaystyle Pleft( frac{-2+6}{2},frac{-2+4}{2} right)=Pleft( 2,1 right).) Абсцисса равна ( displaystyle 2).

Ответ: ( displaystyle 2)

Теперь потренируйся немного самостоятельно, я лишь приведу ответы к каждой задачи, чтобы ты мог себя проверить.

1. Най­ди­те ра­ди­ус окруж­но­сти, опи­сан­ной около тре­уголь­ни­ка, вер­ши­ны ко­то­ро­го имеют ко­ор­ди­на­ты ( displaystyle left( 8;~0 right),~left( 0;~6 right),~left( 8;~6 right).)

2. Най­ди­те ор­ди­на­ту цен­тра окруж­но­сти, опи­сан­ной около тре­уголь­ни­ка, вер­ши­ны ко­то­ро­го имеют ко­ор­ди­на­ты ( displaystyle left( 8;~0 right),~left( 0;~6 right),~left( 8;~6 right).)

3. Ка­ко­го ра­ди­у­са долж­на быть окруж­ность с цен­тром в точке ( displaystyle Pleft( 8;~6 right),) чтобы она ка­са­лась оси абс­цисс?

4. Най­ди­те ор­ди­на­ту точки пе­ре­се­че­ния оси ( displaystyle Oy) и от­рез­ка, со­еди­ня­ю­ще­го точки ( displaystyle Aleft( 6;text{ }8 right)) и ( displaystyle Bleft( -6;text{ }0 right).)

Ответы:

  • ( displaystyle 5);
  • ( displaystyle 3);
  • ( displaystyle 6);
  • ( displaystyle 4).

Умножение векторов

Все удалось? Очень на это надеюсь! Теперь – последний рывок.

Сейчас будь особенно внимателен. Тот материал, который я сейчас буду объяснять, имеет непосредственное отношение не только к простым задачам на метод координат, но также встречается повсеместно и в задачах повышенной сложности.

Какое из своих обещаний я еще не сдержал?

Вспомни, какие операции над векторами я обещал ввести и какие в конечном счете ввел? Я точно ничего не забыл?

Забыл! Забыл объяснить, что значит умножение векторов.

Есть два способа умножить вектор на вектор. В зависимости от выбранного способа у нас будут получаться объекты разной природы:

  • Скалярное произведение (результат – число);
  • Векторное произведение (результат – вектор).

Векторное произведение выполняется довольно хитро. Как его делать и для чего оно нужно, мы с тобой обсудим чуть позже. А пока мы остановимся на скалярном произведении.

Есть аж два способа, позволяющих нам его вычислить:

  • Через координаты векторов;
  • Через длины векторов и угол между ними.

Как ты догадался, результат должен быть один и тот же! Итак, давай вначале рассмотрим первый способ:

Най­ди­те ска­ляр­ное про­из­ве­де­ние век­то­ров ( displaystyle {vec{a}}) и ( displaystyle {vec{b}})

Справился? Может, и подвох небольшой заметил? Давай проверим:

( displaystyle vec{a}left( 2,6 right)), ( displaystyle vec{b}left( 8,4 right)) – координаты векторов, как в прошлой задаче! Ответ: ( displaystyle 40).

Скалярное произведение через длины векторов и косинус угла между ними

Помимо координатного, есть и другой способ вычислить скалярное произведение, а именно, через длины векторов и косинус угла между ними:

( displaystyle left( vec{a},~vec{b} right)=left| {vec{a}} right|left| {vec{b}} right|coswidehat{vec{a},~vec{b}})

( displaystyle widehat{vec{a},~vec{b}}) – обозначает угол между векторами ( displaystyle {vec{a}}) и ( displaystyle {vec{b}}).

То есть скалярное произведение равно произведению длин векторов на косинус угла между ними.

Зачем же нам эта вторая формула, если у нас есть первая, которая намного проще, в ней по крайней мере нет никаких косинусов?

А нужна она для того, что из первой и второй формулы мы с тобой сможем вывести, как находить угол между векторами!

Пусть ( displaystyle vec{a}left( {{x}_{1}},{{y}_{1}} right),~vec{b}left( {{x}_{2}},{{y}_{2}} right).) Тогда вспоминай формулу для длины вектора!

( displaystyle left| {vec{a}} right|=sqrt{x_{1}^{2}+y_{1}^{2}})

( displaystyle left| {vec{b}} right|=sqrt{x_{2}^{2}+y_{2}^{2}})

Тогда если я подставлю эти данные в формулу скалярного произведения, то я получу:

( displaystyle left( vec{a},~vec{b} right)=sqrt{x_{1}^{2}+y_{1}^{2}}cdot sqrt{x_{2}^{2}+y_{2}^{2}}coswidehat{vec{a},~vec{b}})

Но с другой стороны:

( displaystyle left( vec{a},~vec{b} right)={{x}_{1}}{{x}_{2}}+{{y}_{1}}{{y}_{2}})

Тогда

( displaystyle {{x}_{1}}{{x}_{2}}+{{y}_{1}}{{y}_{2}}=sqrt{x_{1}^{2}+y_{1}^{2}}cdot sqrt{x_{2}^{2}+y_{2}^{2}}coswidehat{vec{a},~vec{b}})

Или

( displaystyle coswidehat{vec{a},~vec{b}}=frac{{{x}_{1}}{{x}_{2}}+{{y}_{1}}{{y}_{2}}}{sqrt{x_{1}^{2}+y_{1}^{2}}cdot sqrt{x_{2}^{2}+y_{2}^{2}}})

Таким образом, что же мы с тобой получили?

У нас теперь есть формула, позволяющая вычислять угол между двумя векторами! Иногда ее для краткости записывают еще и так:

( displaystyle coswidehat{vec{a},~vec{b}}=frac{left( vec{a},~vec{b} right)}{left| {vec{a}} right|left| {vec{b}} right|})

Решение:

1. Эти вектора – наши старые знакомые. Их скалярное произведение мы уже считали и оно было равно ( displaystyle 40).

Координаты у них такие: ( displaystyle vec{a}left( 2,6 right)), ( displaystyle vec{b}left( 8,4 right)). Тогда найдем их длины:

( left| {vec{a}} right|=sqrt{{{2}^{2}}+{{6}^{2}}}=sqrt{40})

( left| {vec{b}} right|=sqrt{{{8}^{2}}+{{4}^{2}}}=sqrt{80})

Тогда ищем косинус между векторами:

( coswidehat{vec{a},~vec{b}}=frac{left( vec{a},~vec{b} right)}{left| {vec{a}} right|left| {vec{b}} right|}=frac{40}{sqrt{40}sqrt{80}}=frac{sqrt{40}sqrt{40}}{sqrt{40}sqrt{80}}=frac{1}{sqrt{2}})

Косинус какого угла равен ( frac{1}{sqrt{2}})? Это угол ( 45{}^circ ).

Ответ: ( 45)

Ну а теперь сам реши вторую задачу, а потом сравним! Я приведу лишь очень краткое решение:

2. ( vec{a}+vec{b}) имеет координаты ( left( 10,10 right)), ( vec{a}-vec{b}) имеет координаты ( left( -6,2 right)).

( left( vec{a}+vec{b},vec{a}-vec{b} right)=-60+20=-40)

( left| vec{a}+vec{b} right|=sqrt{{{10}^{2}}+{{10}^{2}}}=10sqrt{2})

( left| vec{a}-vec{b} right|=sqrt{{{left( -6 right)}^{2}}+{{2}^{2}}}=sqrt{40}).

Пусть ( a) – угол между векторами ( vec{a}+vec{b}) и ( vec{a}-vec{b}), тогда

( cosa=frac{-40}{10sqrt{2}sqrt{40}}=-frac{sqrt{40}}{10sqrt{2}}=-frac{sqrt{20}}{10}=-frac{sqrt{5}}{5})

Ответ: ( -frac{sqrt{5}}{5})

Метод координат (продвинутый уровень)

Мы с тобой продолжаем изучать метод координат. В прошлой части мы вывели ряд важных формул, которые позволяют:

  • Находить координаты вектора;
  • Находить длину вектора (альтернативно: расстояние между двумя точками);
  • Складывать, вычитать векторы. Умножать их на вещественное число;
  • Находить середину отрезка;
  • Вычислять скалярное произведение векторов;
  • Находить угол между векторами.

Конечно, в эти 6 пунктов не укладывается весь координатный метод.

Он лежит в основе такой науки, как аналитическая геометрия, с которой тебе предстоит познакомиться в ВУЗе. Я лишь хочу построить фундамент, который позволит тебе решать задачи ЕГЭ любого уровня сложности!

Этот раздел будет посвящен методу решения тех задач, в которых будет разумно перейти к методу координат. Эта разумность определяется тем, что в задаче требуется найти, и какая фигура дана.

Когда стоит применять метод координат

Итак, я бы стал применять метод координат, если ставятся вопросы:

  • Найти угол между двумя плоскостями;
  • Найти угол между прямой и плоскостью;
  • Найти угол между двумя прямыми;
  • Найти расстояние от точки до плоскости;
  • Найти расстояние от точки до прямой;
  • Найти расстояние от прямой до плоскости;
  • Найти расстояние между двумя прямыми.

Подходящими фигурами для метода координат являются:

  • Куб;
  • Прямоугольный параллелепипед;
  • Прямая призма (треугольная, шестиугольная…);
  • Пирамида (треугольная, четырехугольная, шестиугольная);
  • Тетраэдр (одно и то же, что и треугольная пирамида).

Неподходящими фигурами для метода координат являются тела вращения:

  • шар;
  • цилиндр;
  • конус

По моему опыту, нецелесообразно использовать метод координат для:

  • Нахождения площадей сечений;
  • Вычисления объемов тел.

Однако следует сразу отметить, что три «невыгодные» для метода координат ситуации на практике достаточно редки.

В большинстве же задач он может стать твоим спасителем, особенно если ты не очень силен в трехмерных построениях (которые порою бывают довольно замысловатыми).

Как применять метод координат

Какими являются все перечисленные мною выше фигуры?

Они уже не плоские, как, например, квадрат, треугольник, окружность, а объемные! Соответственно, нам нужно рассматривать уже не двухмерную, а трехмерную систему координат.

Строится она достаточно легко: просто помимо оси абсцисс и ординат, мы введем еще одну ось, ось аппликат. На рисунке схематично изображено их взаимное расположение:

Все они являются взаимно перпендикулярными, пересекаются в одной точке ( displaystyle O), которую мы будем называть началом координат.

Ось абсцисс, как и прежде, будем обозначать ( Ox), ось ординат – ( Oy), а введенную ось аппликат – ( Oz).

Если раньше каждая точка на плоскости характеризовалась двумя числами – абсциссой и ординатой, то каждая точка в пространстве уже описывается тремя числами – абсциссой, ординатой, аппликатой.

Например:

Соответственно абсцисса точки ( displaystyle P) равна ( displaystyle 1), ордината – ( displaystyle 2), а аппликата – ( displaystyle 3).

Иногда абсциссу точки еще называют проекцией точки на ось абсцисс, ординату – проекцией точки на ось ординат, а аппликату – проекцией точки на ось аппликат. Соответственно, если задана точка ( Aleft( x,y,z right)) то, точку с координатами:

( Aleft( x,y,0 right)) называют проекцией точки ( Aleft( x,y,z right)) на плоскость ( Oxy)

( Aleft( x,0,z right)) называют проекцией точки ( Aleft( x,y,z right)) на плоскость ( Oxz)

( Aleft( 0,y,z right)) называют проекцией точки ( Aleft( x,y,z right)) на плоскость ( Oyz)

Встает естественный вопрос: справедливы ли все формулы, выведенные для двухмерного случая, в пространстве?

Ответ утвердительный, они справедливы и имеют тот же самый вид. За маленькой деталью. Я думаю, ты уже сам догадался, за какой именно.

Во все формулы мы должны будем добавить еще один член, отвечающий за ось аппликат.

Формулы метода координат для трехмерных фигур

1. Если заданы две точки: ( Aleft( {{x}_{1}},{{y}_{1}},{{z}_{1}} right)), ( Aleft( {{x}_{2}},{{y}_{2}},{{z}_{2}} right)), то:

  • Координаты вектора ( overrightarrow{AB}): ( overrightarrow{AB}left( {{x}_{2}}-{{x}_{1}},{{y}_{2}}-{{y}_{1}},{{z}_{2}}-{{z}_{1}} right));
  • Расстояние между двумя точками (или длина вектора ( overrightarrow{AB})) ( d=left| overrightarrow{AB} right|=sqrt{{{left( {{x}_{2}}-{{x}_{1}} right)}^{2}}+{{left( {{y}_{2}}-{{y}_{1}} right)}^{2}}+{{left( {{z}_{2}}-{{z}_{1}} right)}^{2}}});
  • Середина ( D) отрезка ( AB) имеет координаты
  • ( Dleft( frac{{{x}_{1}}+{{x}_{2}}}{2},frac{{{y}_{1}}+{{y}_{2}}}{2},frac{{{z}_{1}}+{{z}_{2}}}{2} right)).

2. Если дано два вектора: ( vec{a}left( {{x}_{1}},{{y}_{1}},{{z}_{1}} right)) и ( vec{b}left( {{x}_{2}},{{y}_{2}},{{z}_{2}} right)), то:

  • Их скалярное произведение равно: ( left( vec{a},~vec{b} right)=left| {vec{a}} right|left| {vec{b}} right|cosoverset{}{widehat{vec{a},~vec{b}}},) или ( left( vec{a},~vec{b} right)={{x}_{1}}{{x}_{2}}+{{y}_{1}}{{y}_{2}}+{{z}_{1}}{{z}_{2}});
  • Косинус угла между векторами равен:
  • ( cosoverset{}{widehat{vec{a},~vec{b}}},=frac{left( vec{a},~vec{b} right)}{left| {vec{a}} right|left| {vec{b}} right|}=frac{{{x}_{1}}{{x}_{2}}+{{y}_{1}}{{y}_{2}}+{{z}_{1}}{{z}_{2}}}{sqrt{x_{1}^{2}+y_{1}^{2}+z_{1}^{2}}cdot sqrt{x_{2}^{2}+y_{2}^{2}+z_{2}^{2}}}).

Плоскость — как «обобщение» прямой

Однако с пространством не все так просто.

Как ты понимаешь, добавление еще одной координаты вносит существенное разнообразие в спектр фигур, «живущих» в этом пространстве. И для дальнейшего повествования мне потребуется ввести некоторое, грубо говоря, «обобщение» прямой.

Этим «обобщением» будет плоскость. Что ты знаешь про плоскость? Попробуй ответить на вопрос, а что такое плоскость? Очень сложно сказать.

Однако мы все интуитивно представляем, как она выглядит:

Грубо говоря, это некий бесконечный «лист», засунутый в пространство. «Бесконечность» следует понимать, что плоскость распространяется во все стороны, то есть ее площадь равна бесконечности.

Однако, это объяснение «на пальцах» не дает ни малейшего представления о структуре плоскости. А нас будет интересовать именно она.

Давай вспомним одну из основных аксиом геометрии: через две различные точки на плоскости проходит прямая, притом только одна.

Или ее аналог в пространстве: через три точки, не лежащие на одной прямой, проходит плоскость, притом только одна.

Уравнение прямой в плоскости и пространстве

Конечно, ты помнишь, как по двум заданным точкам вывести уравнение прямой, это совсем нетрудно: если первая точка имеет координаты: ( Aleft( {{x}_{0}},{{y}_{0}} right)) а вторая ( Bleft( {{x}_{1}},{{y}_{1}} right)), то уравнение прямой будет следующим:

( frac{x-{{x}_{0}}}{{{x}_{1}}-{{x}_{0}}}=frac{y-{{y}_{0}}}{{{y}_{1}}-{{y}_{0}}})

( left( x-{{x}_{0}} right)left( {{y}_{1}}-{{y}_{0}} right)=left( y-{{y}_{0}} right)left( {{x}_{1}}-{{x}_{0}} right))

Это ты проходил еще в 7 классе.

В пространстве уравнение прямой выглядит вот так: пусть у нас даны две точки с координатами: ( Aleft( {{x}_{0}},{{y}_{0}},{{z}_{0}} right)), ( Bleft( {{x}_{1}},{{y}_{1}},{{z}_{1}} right)), то уравнение прямой, через них проходящей, имеет вид:

( frac{x-{{x}_{0}}}{{{x}_{1}}-{{x}_{0}}}=frac{y-{{y}_{0}}}{{{y}_{1}}-{{y}_{0}}}=frac{z-{{z}_{0}}}{{{z}_{1}}-{{z}_{0}}})

Например, через точки ( Aleft( 1,2,3 right)), ( Bleft( 4,5,6 right)) проходит прямая:

( frac{x-1}{4-1}=frac{y-2}{5-2}=frac{z-3}{6-3})

( frac{x-1}{3}=frac{y-2}{3}=frac{z-3}{3})

( x-1=y-2=z-3)

Как это следует понимать?

Это следует понимать вот как: точка ( Dleft( x,y,z right)) лежит на прямой, если ее координаты удовлетворяют следующей системе:

( displaystyle left{ begin{array}{l}x-1=y-2\x-1=z-3end{array} right.)

Нас не очень будет интересовать уравнение прямой, но нам нужно обратить внимание на очень важное понятие направляющего вектора прямой.

Направляющий вектор прямой

Направляющий вектор прямой – любой ненулевой вектор, лежащий на данной прямой или параллельный ей.

Например, оба вектора ( overrightarrow{{{M}_{0}}{{M}_{1}}}), ( vec{s}) являются направляющими векторами прямой ( l). Пусть ( Mleft( {{x}_{0}},{{y}_{0}},{{z}_{0}} right)) – точка, лежащая на прямой, а ( vec{p}left( m,n,q right)) – ее направляющий вектор.

Тогда уравнение прямой можно записать в следующем виде:

( frac{x-{{x}_{0}}}{m}=frac{y-{{y}_{0}}}{n}=frac{z-{{z}_{0}}}{p})

Еще раз повторюсь, мне не очень будет интересно уравнение прямой, но мне очень нужно, чтобы ты запомнил, что такое направляющий вектор!

Еще раз: это ЛЮБОЙ ненулевой вектор, лежащий на прямой, или параллельный ей.

Уравнение плоскости

Вывести уравнение плоскости по трем заданным точкам уже не так тривиально, и обычно этот вопрос не рассматривается в курсе средней школы.

А зря!

Этот прием жизненно необходим, когда мы прибегаем к методу координат для решения сложных задач. Однако, я предполагаю, что ты полон желания научиться чему-то новому?

Более того, ты сможешь поразить своего преподавателя в ВУЗе, когда выяснится, что ты уже умеешь с методикой, которую обычно изучают в курсе аналитической геометрии. Итак, приступим.

Уравнение плоскости не слишком отличается от уравнения прямой на плоскости, а именно оно имеет вид:

( Ax+By+Cz+D=0)

( A,B,C,D-) некоторые числа (не все равные нулю), а ( x,y,z-~) переменные, например: ( 3x+2y-z+1=0,~0.5x-2z-2=0,~x+y=0) и т.д.

Как видишь, уравнение плоскости не очень отличается от уравнения прямой (линейной функции). Однако, вспомни, что мы с тобой утверждали? Мы говорили, что если у нас есть три точки ( Aleft( {{x}_{0}},{{y}_{0}},{{z}_{0}} right),~Bleft( {{x}_{1}},{{y}_{1}},{{z}_{1}} right),~Cleft( {{x}_{2}},{{y}_{2}},{{z}_{2}} right)), не лежащие на одной прямой, то уравнение плоскости однозначно по ним восстанавливается.

Но как? Попробую тебе объяснить.

Поскольку уравнение плоскости имеет вид:

( Ax+By+Cz+D=0)

А точки ( Aleft( {{x}_{0}},{{y}_{0}},{{z}_{0}} right),~Bleft( {{x}_{1}},{{y}_{1}},{{z}_{1}} right),~Cleft( {{x}_{2}},{{y}_{2}},{{z}_{2}} right)) принадлежат этой плоскости, то при подстановке координат каждой точки в уравнение плоскости мы должны получать верное тождество:

( A{{x}_{0}}+B{{y}_{0}}+C{{z}_{0}}+D=0)

( A{{x}_{1}}+B{{y}_{1}}+C{{z}_{1}}+D=0)

( A{{x}_{2}}+B{{y}_{2}}+C{{z}_{2}}+D=0)

Таким образом, встает необходимость решать три уравнения аж с ( displaystyle 4) неизвестными!

Дилемма! Однако всегда можно предполагать, что ( D=1) (для этого нужно разделить ( ~Ax+By+Cz+D=0) на ( D)).

Таким образом, мы получим три уравнения с тремя неизвестными ( displaystyle A,B,C):

( A{{x}_{0}}+B{{y}_{0}}+C{{z}_{0}}+1=0)

( A{{x}_{1}}+B{{y}_{1}}+C{{z}_{1}}+1=0)

( A{{x}_{2}}+B{{y}_{2}}+C{{z}_{2}}+1=0)

Однако мы не будем решать такую систему, а выпишем загадочное выражение, которое из него следует:

Уравнение плоскости, проходящей через три заданные точки

(left| {begin{array}{*{20}{c}}{x — {x_0}}&{{x_1} — {x_0}}&{{x_2} — {x_0}}\{y — {y_0}}&{{y_1} — {y_0}}&{{y_2} — {y_0}}\{z — {z_0}}&{{z_1} — {z_0}}&{{z_2} — {z_0}}end{array}} right| = 0)

Стоп! Это еще что такое? Какой-то очень необычный модуль!

Однако объект, который ты видишь перед собой не имеет ничего общего с модулем. Этот объект называется определителем третьего порядка.

Определитель третьего порядка

Отныне и впредь, когда ты будешь иметь дело с методом координат на плоскости, тебе очень часто будут встречаться эти самые определители.

Что же такое определитель третьего порядка? Как ни странно, это всего-навсего число. Осталось понять, какое конкретно число мы будем сопоставлять с определителем.

Давай вначале запишем определитель третьего порядка в более общем виде:

( left| {begin{array}{*{20}{c}}{{a_{11}}}&{{a_{12}}}&{{a_{13}}}\{{a_{21}}}&{{a_{22}}}&{{a_{23}}}\{{a_{31}}}&{{a_{32}}}&{{a_{33}}}end{array}} right|),

Где ( {{a}_{ij}}) – некоторые числа.

Причем под первым индеком ( displaystyle i) мы понимаем номер строки, а под индеком ( displaystyle j) – номер столбца.

Например, ( {{a}_{23}}) означает, что данное число стоит на пересечении второй строки и третьего столбца.

Давай поставим следующий вопрос: каким именно образом мы будем вычислять такой определитель?

То есть, какое конкретно число мы будем ему сопоставлять?

Для определителя именно третьего порядка есть эвристическое (наглядное) правило треугольника оно выглядит следующим образом:

Как его читать? А понимать его надо следующим образом: мы составляем два выражения:

  • Произведение элементов главной диагонали (с верхнего левого угла до нижнего правого) ( displaystyle +) произведение элементов, образующих первый треугольник «перпендикулярный» главной диагонали ( displaystyle +) произведение элементов, образующих второй треугольник «перпендикулярный» главной диагонали;
  • Произведение элементов побочной диагонали (с верхнего правого угла до нижнего левого) ( displaystyle +) произведение элементов, образующих первый треугольник «перпендикулярный» побочной диагонали ( displaystyle +) произведение элементов, образующих второй треугольник «перпендикулярный» побочной диагонали;
  • Тогда определитель равен разности значений, полученных на шаге ( displaystyle 1) и ( displaystyle 2).

Если записать все это цифрами, то мы получим следующее выражение:

( left| {begin{array}{*{20}{c}}{{a_{11}}}&{{a_{12}}}&{{a_{13}}}\{{a_{21}}}&{{a_{22}}}&{{a_{23}}}\{{a_{31}}}&{{a_{32}}}&{{a_{33}}}end{array}} right| = )

( = {a_{11}}{a_{22}}{a_{33}} + {a_{12}}{a_{23}}{a_{31}} + {a_{21}}{a_{32}}{a_{13}} — left( {{a_{13}}{a_{22}}{a_{31}} + {a_{23}}{a_{32}}{a_{11}} + {a_{21}}{a_{12}}{a_{33}}} right))

Тем не менее, запоминать способ вычисления в таком виде не нужно, достаточно в голове просто держать треугольники и саму идею, что с чем складывается и что из чего затем вычитается).

Давай проиллюстрируем метод треугольников на примере:

Метод треугольников на примере

1. Вычислить определитель: ( left| {begin{array}{*{20}{c}}2&3&{ — 1}\{11}&{21}&{ — 5}\4&6&9end{array}} right|)

Давай разбираться, что мы складываем, а что – вычитаем.

Слагаемые, которые идут с «плюсом»:

Это главная диагональ: произведение элементов равно 

( 2cdot 21cdot 9=378)

Первый треугольник, «перпендикулярный главной диагонали: произведение элементов равно 

( 3cdot left( -5 right)cdot 4=-60)

Второй треугольник, «перпендикулярный главной диагонали: произведение элементов равно 

( 11cdot 6cdot left( -1 right)=-66)

Складываем три числа: ( 378-60-66=252)

Слагаемые, которые идут с «минусом»:

Это побочная диагональ: произведение элементов равно 

( left( -1 right)cdot 21cdot 4=-84)

Первый треугольник, «перпендикулярный побочной диагонали: произведение элементов равно 

( 3cdot 11cdot 9=297)

Второй треугольник, «перпендикулярный побочной диагонали: произведение элементов равно 

( 6cdot left( -5 right)cdot 2=-60)

Складываем три числа:

( -84+297-60=153)

Все, что осталось сделать – это вычесть из суммы слагаемых «с плюсом» сумму слагаемых «с минусом»:

( 252-153=99)

Таким образом,

( left| {begin{array}{*{20}{c}}2&3&{ — 1}\{11}&{21}&{ — 5}\4&6&9end{array}} right| = 99)

Как видишь, ничего сложного и сверхъестественного в вычислении определителей третьего порядка нет. Просто важно помнить про треугольники и не допускать арифметических ошибок. 

Теперь попробуй самостоятельно вычислить:

( left| {begin{array}{*{20}{c}}2&{ — 2}&4\3&2&5\1&2&2end{array}} right|)

Проверяем:

  • Главная диагональ: ( 2cdot 2cdot 2=8);
  • Первый треугольник, перпендикулярный главной диагонали: ( left( -2 right)cdot 5cdot 1=-10);
  • Второй треугольник, перпендикулярный главной диагонали: ( 3cdot 2cdot 4=24);
  • Сумма слагаемых с плюсом: ( 8-10+24=22);
  • Побочная диагональ: ( 1cdot 2cdot 4=8);
  • Первый треугольник, перпендикулярный побочной диагонали: ( 2cdot 5cdot 2=20);
  • Второй треугольник, перпендикулярный побочной диагонали: ( left( -2 right)cdot 3cdot 2=-12);
  • Сумма слагаемых с минусом: ( 8+20-12=16);
  • Сумма слагаемых с плюсом минус сумма слагаемых с минусом: ( 22-16=6).

Вывод:

( left| {begin{array}{*{20}{c}}2&{ — 2}&4\3&2&5\1&2&2end{array}} right| = 6)

Вот тебе еще пара определителей, вычисли их значения самостоятельно и сравни с ответами:

  • ( left| {begin{array}{*{20}{c}}1&3&{ — 1}\0&4&2\{ — 3}&2&0end{array}} right|);
  • ( left| {begin{array}{*{20}{c}}3&1&7\6&2&{14}\{ — 1}&0&8end{array}} right|).

Ответы:

  • ( displaystyle -34);
  • ( displaystyle 0).

Ну что, все совпало?

Отлично, тогда можно двигаться дальше! Если же есть затрудения, то совет мой таков: в интернете есть куча программ вычисления определителя онлайн.

Все, что тебе нужно – придумать свой определитель, вычислить его самостоятельно, а потом сравнить с тем, что посчитает программа.

И так до тех пор, пока результаты не начнут совпадать. Уверен, этот момент не заставит себя долго ждать!

Теперь давай вернемся к тому определителю, который я выписал, когда говорил про уравнение плоскости, проходящей через три заданные точки:

( left| {begin{array}{*{20}{c}}{x — {x_0}}&{{x_1} — {x_0}}&{{x_2} — {x_0}}\{y — {y_0}}&{{y_1} — {y_0}}&{{y_2} — {y_0}}\{z — {z_0}}&{{z_1} — {z_0}}&{{z_2} — {z_0}}end{array}} right| = 0)

Все, что тебе нужно – это вычислить его значение непосредственно (методом треугольников) и приравнять результат к нулю.

Естественно, поскольку ( displaystyle x,y,z) – переменные, то ты получишь некоторое выражение, от них зависящее.

Именно это выражение и будет уравнением плоскости, проходящей через три заданные точки, не лежащие на одной прямой!

( Ax+By+Cz+D=0)

Давай проиллюстрируем сказанное на простом примере:

1. Построить уравнение плоскости, проходящей через точки

( displaystyle {{M}_{1}}left( -3,2,-1 right), {{M}_{2}}left( -1,2,4 right), {{M}_{3}}left( 3,3,-1 right))

Cоставляем для этих трех точек определитель:

( left| {begin{array}{*{20}{c}}{x — left( { — 3} right)}&{ — 1 — left( { — 3} right)}&{3 — left( { — 3} right)}\{y — 2}&{2 — 2}&{3 — 2}\{z — left( { — 1} right)}&{4 — left( { — 1} right)}&{ — 1 — left( { — 1} right)}end{array}} right|).

Упрощаем:

( left| {begin{array}{*{20}{c}}{x + 3}&2&6\{y — 2}&0&1\{z + 1}&5&0end{array}} right|)

Теперь вычисляем его непосредственно по правилу треугольников:

[{left| {begin{array}{*{20}{c}}{x + 3}&2&6\{y — 2}&0&1\{z + 1}&5&0end{array}} right| = left( {x + 3} right) cdot 0 cdot 0 + 2 cdot 1 cdot left( {z + 1} right) + left( {y — 2} right) cdot 5 cdot 6 — }]

( displaystyle -left( left( z+1 right)cdot 6cdot 0+left( x+3 right)cdot 5cdot 1+left( y-2 right)cdot 2cdot 0 right)=)

( displaystyle=2left( z-1 right)+30left( y-2 right)-5left( x+3 right)=-5x+30y+2z-73)

Таким образом, уравнение плоскости, проходящей через точки ( displaystyle {{M}_{1}}left( -3,2,-1 right), {{M}_{2}}left( -1,2,4 right), {{M}_{3}}left( 3,3,-1 right)), имеет вид:

( -5x+30y+2z-73=0)

То есть ( A=-5,~B=30,~C=2,~D=-73)

Теперь попробуй решить одну задачку самостоятельно, а потом мы ее обсудим:

2. Найти уравнение плоскости, проходящей через точки

( {{M}_{1}}left( 1,2,-1 right),~{{M}_{2}}left( -1,0,4 right),~{{M}_{3}}left( -2,-1,1 right))

Ну что, давай теперь обсудим решение:

Составляем определитель:

( left| {begin{array}{*{20}{c}}{x — 1}&{ — 2}&{ — 3}\{y — 2}&{ — 2}&{ — 3}\{z + 1}&5&2end{array}} right|)

И вычисляем его значение:

( begin{array}{l}left| {begin{array}{*{20}{c}}{x — 1}&{ — 2}&{ — 3}\{y — 2}&{ — 2}&{ — 3}\{z + 1}&5&2end{array}} right| = \ = — 4left( {x — 1} right) — 15left( {y — 2} right) + 6left( {z + 1} right) + 15left( {x — 1} right) + 4left( {y — 2} right) — 6left( {z + 1} right) = \ = 11x — 11y + 11end{array})

Тогда уравнение плоскости имеет вид:

( 11x-11y+11=0)

Или же, сократив на ( 11), получим:

( x-y+1=0)

То есть, ( A=1,B=-1,C=0,D=1.)

Теперь две задачи для самоконтроля:

  • Построить уравнение плоскости, проходящей через три точки: ( Kleft( 2,3,4 right),~Lleft( 6,-3,4 right),~Mleft( -4,6,-4 right).);
  • Построить уравнение плоскости, проходящей через три точки:
  • ( Aleft( 5,-1,3 right),~Bleft( 2,2,0 right),~Cleft( -1,1,1 right).).

Проверим:

  • ( 6x+4y-3z-12=0);
  • ( y+z-2=0).

Все совпало?

Опять-таки, если есть определенные затруднения, то мой совет таков: берешь из головы три точки (с большой степенью вероятности они не будут лежать на одной прямой), строишь по ним плоскость.

А потом проверяешь себя онлайн. Например, на сайте:

http://www.webmath.ru/web/prog9_1.php

Однако при помощи определителей мы будем строить не только уравнение плоскости. 

Вспомни, я говорил тебе, что для векторов определено не только скалярное произведение. Есть еще векторное, а также смешанное произведение.

Векторное произведение векторов

И если скалярным произведением двух векторов и будет число, то векторным произведением двух векторов ( vec{a}) и ( vec{b}) будет вектор ( ~vec{c}=vec{a}cdot vec{b}), причем данный вектор будет перпендикулярен к заданным:

Причем его модуль будет равен площади параллелограмма, построенного на векторах ( vec{a}) и ( vec{b}).

Данный вектор понадобится нам для вычисления расстояния от точки до прямой. Как же нам считать векторное произведение векторов ( vec{a}) и ( vec{b}), если их координаты заданы?

На помощь к нам опять приходит определитель третьего порядка.

Однако, прежде чем я перейду к алгоритму вычисления векторного произведения, я вынужден сделать небольшое лирическое отступление.

Данное отступление касается базисных векторов.

Базисными векторами в трехмерном пространстве называются три вектора:

( vec{i}left( 1,0,0 right),~vec{j}left( 0,1,0 right),~vec{k}left( 0,0,1 right))

Схематично они изображены на рисунке:

Как ты думаешь, а почему они называется базисными? Дело в том, что любой вектор в трехмерном пространстве можно представить через сумму трех базисных векторов:

( vec aleft( {x,y,z} right) = x cdot vec i + y cdot vec j + z cdot vec k.)

Или на картинке:

Справедливость этой формулы очевидна, ведь:

( begin{array}{l}xcdot vec{i}=left( x,0,0 right)\ycdot vec{j}=left( 0,y,0 right)\zcdot vec{k}=left( 0,0,z right)end{array})

Тогда

( vec{a}left( x,y,z right)=xcdot vec{i}+ycdot vec{j}+zcdot vec{k}=left( x,0,0 right)+left( 0,y,0 right)+left( 0,0,z right)=left( x,y,z right)=vec{a}.)

Смешанное произведение трех векторов

Последняя конструкция, которая мне понадобится – это смешанное произведение трех векторов. 

Оно, как и скалярное, является числом. Есть два способа его вычисления. ( displaystyle 1) – через определитель, ( displaystyle 2) – через смешанное произведение.

А именно, пусть у нас даны три вектора:

( vec{a}left( {{x}_{1}},{{y}_{1}},{{z}_{1}} right),~vec{b}left( {{x}_{2}},{{y}_{2}},{{z}_{2}} right),~vec{c}left( {{x}_{3}},{{y}_{3}},{{z}_{3}} right)), тогда смешанное произведение трех векторов, обозначаемое через ( (vec{a},vec{b},vec{c})) можно вычислить как:

1. ( left( vec{a},vec{b},vec{c} right)=left( vec{a},vec{b}cdot vec{c} right)) – то есть смешанное произведение – это скалярное произведения вектора на векторное произведение двух других векторов

2. ( left( {vec a,vec b,vec c} right) = left| {begin{array}{*{20}{c}}{{x_1}}&{{y_1}}&{{z_1}}\{{x_2}}&{{y_2}}&{{z_2}}\{{x_3}}&{{y_3}}&{{z_3}}end{array}} right|)

Например, смешанное произведение трех векторов ( vec{a}left( 2,3,5 right),~vec{b}left( 1,4,4 right),~vec{c}left( 3,5,7 right)) равно:

( left( {vec a,vec b,vec c} right) = left| {begin{array}{*{20}{c}}2&3&5\1&4&4\3&5&7end{array}} right| = — 4)

Самостоятельно попробуй вычислить его через векторное произведение и убедись, что результаты совпадут!

И опять – два примера для самостоятельного решения:

  • ( vec{a}left( 1,2,3 right),~vec{b}left( 1,1,1 right),~vec{c}left( 1,2,1 right));
  • ( vec{a}left( 1,2,3 right),~vec{b}left( 1,-1,1 right),~vec{c}left( 2,0,-1 right)).

Ответы:

  • ( displaystyle 2);
  • ( displaystyle 1).

Выбор системы координат

Ну вот, теперь у нас есть весь необходимый фундамент знаний, чтобы решать сложные стереометрические задачи по геометрии.

Однако прежде чем приступать непосредственно к примерам и алгоритмам их решения, я считаю, что будет полезно остановиться еще вот на каком вопросе: как именно выбирать систему координат для той или иной фигуры.

Ведь именно выбор взаимного расположения системы координат и фигуры в пространстве в конечном счете определит, насколько громоздкими будут вычисления.

Я напомню, что в этом разделе мы рассматриваем следующие фигуры:

  • куб;
  • Прямоугольный параллелепипед;
  • Прямая призма (треугольная, шестиугольная…);
  • Пирамида (треугольная, четырехугольная);
  • Тетраэдр (одно и то же, что и треугольная пирамида).

Для каждой из фигур я дам практические рекомендации, как выбирать систему координат.

Я неслучайно расположил задачи в таком порядке. Пока ты еще не успел начать ориентироваться в методе координат, я сам разберу наиболее «проблемные» фигуры, а тебе предоставлю разобраться с простейшим кубом!

Постепенно тебе предстоит научиться работать со всеми фигурами, сложность задач я буду увеличивать от теме к теме.

Приступаем к решению задач:

1. Рисуем тетраэдр, помещаем его в систему координат так, как я предлагал ранее. Поскольку тетраэд правильный – то все его грани (включая основание) – правильные треугольники.

Поскольку нам не дана длина стороны, то я могу принять ее равной ( 1). Я думаю, ты понимаешь, что угол на самом деле не будет зависеть от того, насколько наш тетраэдр будет «растянут»?

Также проведу в тетраэдре высоту и медиану ( displaystyle BM).

Попутно я нарисую его основание (оно нам тоже пригодится).

Мне нужно найти угол между ( displaystyle DH) и ( displaystyle BM). Что нам известно?

Нам известна только координата точки ( displaystyle B). Значит, надо найти еще координаты точек ( displaystyle D,H,M).

Теперь думаем: точка ( displaystyle H) – это точка пересечения высот (или биссектрисс или медиан) треугольника ( displaystyle ABC).

А точка ( displaystyle D) – это приподнятая точка ( displaystyle H).

Точка же ( displaystyle M) – это середина отрезка ( displaystyle AD).

Тогда окончательно нам надо найти: координаты точек: ( displaystyle A,D,H,M).

Начнем с самого простого: координаты точки ( displaystyle A).

Смотри на рисунок: Ясно, что аппликата точки ( displaystyle A) равна нулю (точка лежит на плоскости ( displaystyle Oxy)).

Её ордината равна ( displaystyle 0,5) (так как ( displaystyle AK) – медиана).

Сложнее найти ее абсциссу. Однако это легко делается на основании теоремы Пифагора: Рассмотрим треугольник ( displaystyle BAS). Его гипотенуза ( displaystyle BA) равна ( displaystyle 1), а один из катетов ( displaystyle AS) равен ( displaystyle 0,5)

Тогда:

( BS=sqrt{B{{A}^{2}}-A{{S}^{2}}}=sqrt{1-frac{1}{4}}=frac{sqrt{3}}{2})

Окончательно имеем: ( Aleft( frac{sqrt{3}}{2},frac{1}{2},0 right)).

Теперь найдем координаты точки ( displaystyle H).

Ясно, что ее аппликата опять равна нулю, а ее ордината такая же, как у точки ( displaystyle A), то есть ( 0,5).

Найдем ее абсциссу. Это делается достаточно тривиально, если помнить, что высоты равностороннего треугольника точкой пересечения делятся в пропорции ( displaystyle mathbf{2}:mathbf{1}), считая от вершины. Так как: ( AK=BS=frac{sqrt{3}}{2}), то искомая абсцисса точки, равная длине отрезка ( displaystyle KH), равна: ( KH=frac{AK}{3}=frac{sqrt{3}}{6}). Т

аким образом, координаты точки ( displaystyle H) равны:

( Hleft( frac{sqrt{3}}{6},frac{1}{2},0 right).)

Найдем координаты точки ( displaystyle D).

Ясно, что ее абсцисса и ордината совпадают с абсциссой и ординатой точки ( displaystyle H). А аппликата равна длине отрезка ( displaystyle DH). ( displaystyle DH) – это один из катетов треугольника ( displaystyle DAH). Гипотенуза треугольника ( displaystyle DAH) – это отрезок ( AD=AB=1.) ( displaystyle AH) – катет.

Он ищется из соображений, которые я выделил жирным шрифтом:

( AH=frac{2}{3}cdot frac{sqrt{3}}{2}=frac{sqrt{3}}{3})

Тогда:

( DH=sqrt{1-{{left( frac{sqrt{3}}{3} right)}^{2}}}=sqrt{frac{2}{3}})

Отсюда:

( Dleft( frac{sqrt{3}}{6},frac{1}{2},sqrt{frac{2}{3}} right).)

Точка ( M) – это середина отрезка ( AD). Тогда нам нужно вспомнить формулу координат середины отрезка:

( Mleft( frac{frac{sqrt{3}}{2}+frac{sqrt{3}}{6}}{2},~frac{frac{1}{2}+frac{1}{2}}{2},frac{0+sqrt{frac{2}{3}}}{2} right)=Mleft( frac{sqrt{3}}{3},frac{1}{2},frac{1}{sqrt{6}} right).~)

Ну все, теперь мы можем искать координаты направляющих векторов:

( overrightarrow{BM}left( frac{sqrt{3}}{3},frac{1}{2},frac{1}{sqrt{6}} right))

( overrightarrow{DH}left( 0,0,-sqrt{frac{2}{3}} right))

Ну что, все готово: подставляем все данные в формулу:

( displaystyle cosvarphi =frac{left| frac{1}{sqrt{6}}cdot left( -sqrt{frac{2}{3}} right) right|}{sqrt{{{left( frac{sqrt{3}}{3} right)}^{2}}+{{left( frac{1}{2} right)}^{2}}+{{left( frac{1}{sqrt{6}} right)}^{2}}}cdot sqrt{{{left( -sqrt{frac{2}{3}} right)}^{2}}}}=frac{frac{1}{3}}{sqrt{frac{19}{36}}cdot sqrt{frac{2}{3}}}=frac{frac{1}{3}}{sqrt{frac{19}{54}}}=frac{sqrt{54}}{3sqrt{19}}=sqrt{frac{6}{19}})

Таким образом, ( varphi =arccossqrt{frac{6}{19}}.)

Ответ: ( varphi =arccossqrt{frac{6}{19}}.)

Тебя не должны пугать такие «страшные» ответы: для задач С2 это обычная практика. Я бы скорее удивился «красивому» ответу в этой части. Также, как ты заметил, я практически не прибегал ни к чему, кроме как к теореме Пифагора и свойству высот равностороннего треугольника. То есть для решения стереометрической задачи я использовал самый минимум стереометрии. Выигрыш в этом частично «гасится» достаточно громоздкими вычислениями. Зато они достаточно алгоритмичны!

2. Изобразим правильную шестиугольную пирамиду вместе с системой координат, а также ее основание:

Нам нужно найти угол между прямыми ( displaystyle SB) и ( displaystyle CD).

Таким образом, наша задача сводится к поиску координат точек: ( displaystyle S,B,C,D).

Координаты последних трех мы найдем по маленькому рисунку, а коодинату вершины ( displaystyle S) найдем через координату точки ( displaystyle O).

Работы навалом, но надо к ней приступать!

a) Координата ( displaystyle D): ясно, что ее аппликата и ордината равны нулю.

Найдем абсциссу. Для этого рассмотрим прямоугольный треугольник ( displaystyle EDP). Увы, в нем нам известна только гипотенуза, которая равна ( displaystyle 1). Катет ( displaystyle DP) мы будем стараться отыскать (ибо ясно, что удвоенная длина катета ( displaystyle DP) даст нам абсциссу точки ( displaystyle D)).

Как же нам ее искать?

Давай вспомним, что за фигура у нас лежит в основании пирамиды? Это правильный шестиугольник.

А что это значит? Это значит, что у него все стороны и все углы равны. Надо бы найти один такой угол. Есть идеи?

Идей масса, но есть формула:

Сумма углов правильного n-угольника равна ( left( n-2 right)cdot 180{}^circ ).

Таким образом, сумма углов правильного шестиугольника равна ( displaystyle 720) градусов. Тогда каждый из углов равен:

( frac{720{}^circ }{6}=120{}^circ )

Вновь смотрим на картинку.

Ясно, что отрезок ( displaystyle EB) – биссектрисса угла ( displaystyle DEF). Тогда угол ( displaystyle DEP) равен ( displaystyle 60) градусам.

Тогда:

( sin60{}^circ =frac{sqrt{3}}{2}=frac{DP}{ED}=frac{DP}{1}=DP)

Тогда ( DP=frac{sqrt{3}}{2}), откуда ( DF=2DP=sqrt{3}).

Таким образом, ( displaystyle D) имеет координаты ( Dleft( sqrt{3},0,0 right))

b) Теперь легко найдем координату точки ( C): ( Cleft( sqrt{3},1,0 right)).

c) Найдем координаты точки ( displaystyle B).

Так как ее абсцисса совпадает с длиной отрезка ( FP) то она равна ( frac{sqrt{3}}{2}).

Найти ординату тоже не очень сложно: если мы соединим точки ( displaystyle C) и ( displaystyle A) а точку пересечения прямой ( displaystyle AC) обозначим, скажем за ( displaystyle M). (сделай сам несложное построение). Тогда ( BM=EP.)

Таким образом, ордината точки B равна сумме длин отрезков ( PM+MB). Вновь обратимся к треугольнику ( displaystyle DEP).

Тогда

( frac{1}{2}=cos60{}^circ =frac{EP}{ED}=EP)

Тогда так как ( PM=DC=1,~mo~PB=1+frac{1}{2}=frac{3}{2}.) Тогда точка ( B) имеет координаты ( Bleft( frac{sqrt{3}}{2},frac{3}{2},0 right).)

d) Теперь найдем координаты точки ( displaystyle O).

Рассмотри прямоугольник ( displaystyle ACDF) и докажи, что ( PO=frac{1}{2}.)

Таким образом, координаты точки ( displaystyle O): ( Oleft( frac{sqrt{3}}{2},frac{1}{2},0 right).)

e) Осталось найти координаты вершины ( S). Ясно, что ее абсцисса и ордината совпадает с абсциссой и ординатой точки ( O).

Найдем аппликату. Так как ( FC=EB=2), то ( OF=1). Рассмотрим прямоугольный треугольник ( displaystyle OFS). По условию задачи боковое ребро ( FS=2). Это гипотенуза моего треугольника.

Тогда высота пирамиды ( displaystyle OS) – катет.

( OS=sqrt{F{{S}^{2}}-O{{F}^{2}}}=sqrt{4-1}=sqrt{3})

Тогда точка ( S) имеет координаты: ( Sleft( frac{sqrt{3}}{2},frac{1}{2},sqrt{3} right).)

Ну все, у меня есть координаты всех интересующих меня точек. Ищу координаты направляющих векторов прямых:

( overrightarrow{SB}left( frac{sqrt{3}}{2}-frac{sqrt{3}}{2},frac{1}{2}-frac{3}{2},sqrt{3}-0 right)=overrightarrow{SB}left( 0,-1,sqrt{3} right).)

( overrightarrow{CD}left( sqrt{3}-sqrt{3},0-1,0 right)=overrightarrow{CD}left( 0,-1,0 right).)

Ищем угол между этими векторами:

( cosvarphi =frac{left| 0+left( -1 right)cdot left( -1 right)+sqrt{3}cdot 0 right|}{sqrt{{{left( -1 right)}^{2}}+{{left( sqrt{3} right)}^{2}}}cdot sqrt{{{left( -1 right)}^{2}}}}=frac{1}{2})

Тогда ( varphi =arccos left( frac{1}{2} right)=60{}^circ )

Ответ: ( 60{}^circ )

Опять-таки, при решении этой задачи я не использовал никаких изошренных приемов, кроме формулы суммы углов правильного n-угольника, а также определения косинуса и синуса прямоугольного треугольника.

3. Поскольку нам опять не даны длины ребер в пирамиде, то я буду считать их равными единице. 

Таким образом, поскольку ВСЕ ребра, а не только боковые, равны между собой, то в основании пирамиды и меня лежит квадрат, а боковые грани – правильные треугольники.

Изобразим такую пирамиду, а также ее основание на плоскости, отметив все данные, приведенные в тексте задачи:

Ищем угол между ( displaystyle BM) и ( displaystyle PH).

Я буду делать очень краткие выкладки, когда буду заниматься поиском координат точек. Тебе необходимо будет «расшифровать» их:

a) ( Bleft( 0,1,0 right))

b) ( displaystyle H) – середина отрезка ( displaystyle AC). Её координаты:

( Hleft( frac{1}{2},frac{1}{2},0 right))

c) Длину отрезка ( displaystyle AH) я найду по теореме Пифагора в треугольнике ( displaystyle AHD). ( AH=frac{sqrt{2}}{2}.) Найду ( displaystyle PH) по теореме Пифагора в треугольнике ( displaystyle AHP).

( PH=sqrt{1-frac{1}{2}}=frac{1}{sqrt{2}})

Координаты ( P): ( Pleft( frac{1}{2},frac{1}{2},frac{1}{sqrt{2}} right).)

d) ( M) – середина отрезка ( AP). Ее координаты равны ( Mleft( frac{1}{4},frac{1}{4},frac{1}{2sqrt{2}} right).)

e) Координаты вектора ( overrightarrow{PH}:~overrightarrow{PH}left( 0,0,-frac{1}{sqrt{2}} right).~)

f) Координаты вектора ( overrightarrow{BM}:~overrightarrow{BM}left( frac{1}{4},-frac{3}{4},frac{1}{2sqrt{2}} right).)

g) Ищем угол: ( cosvarphi =frac{frac{1}{4}}{frac{1}{sqrt{2}}cdot frac{sqrt{3}}{2}}=frac{1}{sqrt{6}})

h) Ответ: ( arccosfrac{1}{sqrt{6}})

Куб – простейшая фигура. Я уверен, что с ней ты разберешься самостоятельно. Ответы к задачам 4 и 5 следующие:

4. ( arccosfrac{4}{sqrt{30}})

5. ( arccosfrac{1}{sqrt{15}})

Нахождение угла между прямой и плоскостью

Ну что, время простых задачек окончено!

Теперь примеры будут еще сложнее. Для отыскания угла между прямой и плоскостью мы будем поступать следующим образом:

  • По трем точкам строим уравнение плоскости: ( Ax+By+Cz+D=0), используя определитель третьего порядка;
  • По двум точкам ищем координаты направляющего вектора прямой: ( vec{s}left( l,m,n right));
  • Применяем формулу для вычисления угла между прямой и плоскостью: ( sinvarphi =frac{left| Al+Bm+Cn right|}{sqrt{{{A}^{2}}+{{B}^{2}}+{{C}^{2}}~}cdot sqrt{{{l}^{2}}+{{m}^{2}}+{{n}^{2}}}})

Как видишь, эта формула очень похожа на ту, что мы применяли для поиска углов между двумя прямыми.

Структура правой части просто одинакова, а слева мы теперь ищем синус, а не косинус, как раньше. Ну и добавилось одно противное действие – поиск уравнения плоскости.

Опять я решу первые две задачи подробно, третью – кратко, а последние две оставляю тебе для самостоятельного решения.

К тому же тебе уже приходилось иметь дело с треугольной и четырехугольной пирамидами, а вот с призмами – пока что нет.

Решения:

1. Изобразим призму, а также ее основание. Совместим ее с системой координат и отметим все данные, которые даны в условии задачи:

Извиняюсь за некоторое несоблюдение пропорций, но для решения задачи это, по сути, не так важно. Плоскость ( BC{{C}_{1}}) – это просто «задняя стенка» моей призмы. Достаточно просто догадаться, что уравнение такой плоскости имеет вид:

( x=0)

Однако, это можно показать и непосредственно:

Выберем произвольные три точки на этой плоскости: например, ( Bleft( 0,0,0 right),~Cleft( 0,8,0 right),~{{B}_{1}}left( 0,0,3 right)).

Составим уравнение плоскости:

( left| {begin{array}{*{20}{c}}x&0&0\y&8&0\z&0&3end{array}} right| = 0)

Упражнение тебе: самостоятельно вычислить этот определитель. У тебя получилось ( 24x)? Тогда уравение плоскости имеет вид:

( 24x=0)

Или просто

( x=0)

Таким образом, ( A=1,B=0,C=0,D=0.)

Для решения примера мне нужно найти координаты направляющего вектора прямой ( B{{A}_{1}}).

Так как точка ( B) cовпала с началом координат, то координаты вектора (overrightarrow{B{{A}_{1}}}) просто совпадут с координатами точки ( {{A}_{1}}.)

Для этого найдем вначале координаты точки ( displaystyle A).

Для этого рассмотрим треугольник ( displaystyle ABC).

Проведем высоту (она же – медиана и биссектрисса) из вершины ( displaystyle A).

Так как ( BC=8), то ордината точки ( displaystyle A) равна ( displaystyle 4).

Для того, чтобы найти абсциссу этой точки, нам нужно вычислить длину отрезка ( displaystyle AT).

По теореме Пифагора имеем:

( AT=sqrt{A{{B}^{2}}-B{{T}^{2}}}=sqrt{25-16}=3.)

Тогда точка ( displaystyle A) имеет координаты:

( Aleft( 3,4,0 right))

Точка ( {{A}_{1}})– это «приподнятая» на ( displaystyle 3) точка ( displaystyle A):

( {{A}_{1}}left( 3,4,3 right))

Тогда координаты вектора ( overrightarrow{B{{A}_{1}}}):

( overrightarrow{B{{A}_{1}}}left( 3,4,3 right).)

( sinvarphi =frac{left| 3cdot 1+4cdot 0+3cdot 0 right|}{sqrt{{{1}^{2}}+{{0}^{2}}+{{0}^{2}}}cdot sqrt{{{3}^{2}}+{{4}^{2}}+{{3}^{2}}}}=frac{3}{sqrt{34}}.)

( varphi =arcsinfrac{3}{sqrt{34}}.)

Ответ: ( arcsinfrac{3}{sqrt{34}}.)

Как видишь, ничего принципиально сложного при решении таких задач нет. На самом деле процесс еще немного упрощает «прямота» такой фигуры, как призма.

Теперь давай перейдем к следующему примеру:

2. Рисуем параллелепипед, проводим в нем плоскость и прямую, а также отдельно вычерчиваем его нижнее основание:

Вначале найдем уравнение плоскости: Координаты трех точек, лежащих в ней:

( Aleft( 0,0,0 right),~Bleft( 0,2,0 right),{{C}_{1}}left( 1,2,1 right)) (первые две координаты получены очевидным способом, а последнюю координату ты легко найдешь по картинке из точки ( C)). Тогда составляем уравнение плоскости:

( left| {begin{array}{*{20}{c}}x&0&1\y&2&2\z&0&1end{array}} right| = 0)

Вычисляем:

( 2x-2z=0,~x-z=0)

Тогда ( A=1,B=0,C=-1,D=0.)

Ищем координаты направляющего вектора ( overrightarrow{A{{B}_{1}}}): Ясно, что его координаты совпадают с координатами точки ( {{B}_{1}}), не правда ли?

Как найти координаты ( {{B}_{1}})?

Это же координаты точки ( B), приподнятые по оси аппликат на единицу! ( {{B}_{1}}left( 0,2,1 right)). Тогда ( overrightarrow{A{{B}_{1}}}left( 0,2,1 right).)

Ищем искомый угол:

( sinvarphi =frac{left| 1cdot 0+0cdot 2+left( -1 right)cdot 1 right|}{sqrt{{{1}^{2}}+{{left( -1 right)}^{2}}+{{0}^{2}}~}cdot sqrt{0+{{2}^{2}}+{{1}^{2}}}}=frac{1}{sqrt{10}}.)

( ~varphi =arcsinfrac{1}{sqrt{10}}.)

Ответ: ( arcsinfrac{1}{sqrt{10}}.)

3. Рисуем правильную шестиугольную призму, а затем проводим в ней плоскость и прямую.

Тут даже плоскость нарисовать проблемно, не говоря уже о решении этой задачи, однако методу координат все равно! Именно в его универсальности и заключается его основное преимущество!

Плоскость проходит через три точки: ( A,C,{{D}_{1}}). Ищем их координаты:

1) ( Aleft( 0,0,0 right),~left( frac{sqrt{3}}{2},frac{3}{2},0 right), {{D}_{1}}left( sqrt{3},1,1 right)). Сам выведи координаты для последних двух точек. Тебе пригодится для этого решение задачи с шестиугольной пирамидой!

2) Строим уравнение плоскости:

( left| {begin{array}{*{20}{c}}x&{frac{{sqrt 3 }}{2}}&{sqrt 3 }\y&{frac{3}{2}}&1\z&0&1end{array}} right| = 0)

( -sqrt{3}x+y+2z=0)

( A=-sqrt{3},B=1,C=2,D=0.)

Ищем координаты вектора ( overrightarrow{A{{C}_{1}}}): ( text{ }!!~!!text{ }overrightarrow{A{{C}_{1}}}left( frac{sqrt{3}}{2},frac{3}{2},1 right)). (снова смотри задачу с треугольной пирамидой!)

3) Ищем угол:

( sinvarphi =frac{left| -sqrt{3}cdot frac{sqrt{3}}{2}+frac{3}{2}+2 right|}{sqrt{{{left( frac{sqrt{3}}{2} right)}^{2}}+{{left( frac{3}{2} right)}^{2}}+{{1}^{2}}~}cdot sqrt{{{left( -sqrt{3} right)}^{2}}+{{1}^{2}}+{{2}^{2}}}}=frac{2}{2sqrt{8}}=frac{1}{2sqrt{2}}.)

Ответ: ( arcsinfrac{1}{2sqrt{2}}.)

Как видишь, ничего сверхъестественно сложного в этих задачах нет. Нужно лишь быть очень внимательным с корнями. К последним двум задачам я дам лишь ответы:

4. ( text{arcsin}frac{12}{sqrt{193}}~)

5. ( text{arcsin}frac{1}{sqrt{6}}~)

Как ты мог убедиться, техника решения задач везде одинаковая: основная задача найти координаты вершин и подставить их в некие формулы. Нам осталось рассмотреть еще один класс задач на вычисление углов, а именно: вычисление углов между двумя плоскостями.

Решения задач:

1. Сто­ро­на ос­но­ва­ния пра­виль­ной тре­уголь­ной приз­мы ( ABC{{A}_{1}}{{B}_{1}}{{C}_{1}}) равна ( 2), а диа­го­наль бо­ко­вой грани равна ( sqrt{5}). Най­ди­те угол между плос­ко­стью ( {{A}_{1}}BC) и плос­ко­стью ос­но­ва­ния приз­мы.

Рисую правильную (в основании – равносторонний треугольник) треугольную призму и отмечаю на ней плоскости, которые фигурируют в условии задачи:

Нам нужно найти уравнения двух плоскостей: ( ABC~и~BC{{A}_{1}}.) Уравнение основания получается тривиально: ты можешь составить соответствующий определитель по трем точкам, я же составлю уравнение сразу:

( z=0.)

То есть:

( {{A}_{1}}=0, {{B}_{1}}=0, {{C}_{1}}=1, {{D}_{1}}=0.)

Теперь найдем уравнение ( BC{{A}_{1}}.) Точка ( B) имеет координаты ( Bleft( 0,0,0 right).) Точка ( C) – ( Cleft( 0,1,0 right).)

Так как ( AO) – медиана и высота треугольника ( ABC), то ( BO=OC=1.) ( AO) легко находится по теореме Пифагора в треугольнике ( BAO:) ( AO=sqrt{4-1}=sqrt{3}).

Тогда точка ( A) имеет координаты: ( Aleft( sqrt{3},1,0 right).)

Найдем аппликату точки ( {{A}_{1}}.) Для этого рассмотрим прямоугольный треугольник ( {{A}_{1}}AC.~)

( A{{A}_{1}}=sqrt{{{A}_{1}}{{C}^{2}}-A{{C}^{2}}}=1.)

Тогда получаем вот такие координаты: ( {{A}_{1}}left( sqrt{3},1,1 right).) Cоставляем уравнение плоскости ( BC{{A}_{1}}).

( left| {begin{array}{*{20}{c}}x&0&{sqrt 3 }\y&1&1\z&0&1end{array}} right| = 0.)

( x+sqrt{3}z-sqrt{3}z-sqrt{3}y=0)

( x-sqrt{3}z=0)

Тогда

( {{A}_{2}}=1, {{B}_{2}}=0, {{C}_{2}}=-sqrt{3}, {{D}_{2}}=0.)

Вычисляем угол между плоскостями:

( cosvarphi =frac{left| -sqrt{3} right|}{sqrt{1+{{left( -sqrt{3} right)}^{2}}}}=frac{sqrt{3}}{2}.)

Отсюда

( varphi =30{}^circ .)

Ответ: ( 30{}^circ .)

2. В пра­виль­ной че­ты­рех­уголь­ной пи­ра­ми­де ( displaystyle SABCD), все ребра ко­то­рой равны ( displaystyle 1), най­ди­те синус угла между плос­ко­стью ( displaystyle SAD) и плос­ко­стью, про­хо­дя­щей через точку ( displaystyle A) пер­пен­ди­ку­ляр­но пря­мой ( displaystyle BD).

Делаем рисунок:

Самое сложное – это понять, что это такая за таинственная плоскость, проходящая через точку ( A) перпендикулярно ( DB).

Ну что же, главное, это что? Главное – это внимательность! В самом деле, прямая ( AC) перпендикулярна ( BD). Прямая ( OS) также перпендикулярна ( BD).

Тогда плоскость, проходящая через эти две прямые, будет перпендикулярна прямой ( BD), и, кстати, проходить через точку ( A). Эта плоскость также проходит через вершину пирамиды.

Тогда искомая плоскость – ( SAC.) А плоскость ( SAD) нам уже дана. Ищем координаты точек ( displaystyle S,A,C,D).

  • ( displaystyle Aleft( 0,1,0 right))
  • ( displaystyle Cleft( 1,0,0 right))
  • ( displaystyle Dleft( 0,0,0 right))

Координату точки ( S) найдем через точку ( O). Из маленького рисунка легко вывести, что координаты у точки ( O) будут такие: ( Oleft( frac{1}{2},frac{1}{2},0 right).~)

Что теперь осталось найти, чтобы найти координаты вершины пирамиды?

Еще нужно вычислить ее высоту.

Это делается при помощи все той же теоремы Пифагора: вначале докажи, что ( OB=frac{sqrt{2}}{2}) (тривиально из маленьких треугольничков, образующих квадрат в основании).

Так как по условию ( SB=1), то имеем:

( OS=sqrt{1-{{left( frac{sqrt{2}}{2} right)}^{2}}}=frac{1}{sqrt{2}}.)

Теперь все готово: координаты вершины:

( Sleft( frac{1}{2},frac{1}{2},frac{1}{sqrt{2}} right).~)

Составляем уравнение плоскости ( displaystyle DAS):

( left| {begin{array}{*{20}{c}}x&0&{frac{1}{2}}\y&1&{frac{1}{2}}\z&0&{frac{1}{{sqrt 2 }}}end{array}} right| = 0)

Ты уже спец в вычислении определителей. Без труда ты получишь:

( frac{1}{sqrt{2}}x-frac{1}{2}z=0)

Или иначе (если домножим обе части на корень из двух)

( x-frac{1}{sqrt{2}}z=0.)

Теперь найдем уравнение плоскости ( displaystyle SAC):

( left| {begin{array}{*{20}{c}}{x — 1}&{ — 1}&{ — frac{1}{2}}\y&1&{frac{1}{2}}\z&0&{frac{1}{{sqrt 2 }}}end{array}} right| = 0)

(ты ведь не забыл, как мы получаем уравнение плоскости, правда?

Если ты не понял, откуда взялась эта минус единица, то вернись к определению уравнения плоскости! Просто всегда до этого оказывалось так, что моей плоскости принадлежало начало координат!)

Вычисляем определитель:

( begin{array}{l}frac{x-1}{sqrt{2}}-frac{1}{2}z+frac{1}{2}z+frac{y}{sqrt{2}}=0\frac{x-1}{sqrt{2}}+frac{y}{sqrt{2}}=0\x+y-1=0end{array}).

(Ты можешь заметить, что уравнение плоскости совпало с уравнением прямой, проходящей через точки ( displaystyle A) и ( displaystyle C)! Подумай, почему!)

Теперь вычисляем угол:

( cosvarphi =frac{left| 1+1cdot 0-frac{1}{sqrt{2}}cdot 0 right|}{sqrt{1+{{left( -frac{1}{sqrt{2}} right)}^{2}}}cdot sqrt{{{1}^{2}}+{{1}^{2}}}~~}=frac{1}{sqrt{3}}.)

Нам же нужно найти синус:

( sinvarphi =sqrt{1-{{cos }^{2}}varphi }=sqrt{1-frac{1}{3}}=sqrt{frac{2}{3}}).

Ответ: ( sqrt{frac{2}{3}}.)

3. В правильной че­ты­рех­уголь­ной призме ( ABCD{{A}_{1}}{{B}_{1}}{{C}_{1}}{{D}_{1}}) сто­ро­ны ос­но­ва­ния равны ( displaystyle 1), а бо­ко­вые ребра равны ( displaystyle 5). На ребре ( A{{A}_{1}}) от­ме­че­на точка ( displaystyle E) так, что ( AE:E{{A}_{1}}=2:3). Найдите угол между плос­ко­стя­ми ( ABC) и ( BE{{D}_{1}}.)

Каверзный вопрос: а что такое прямоугольная призма, как ты думаешь? Это же всего-то навсего хорошо известный тебе параллелепипед! Сразу же делаем чертеж! Можно даже отдельно не изображать основание, пользы от него здесь немного:

Плоскость ( ABC), как мы уже раньше заметили, записывается в виде уравнения:

( z=0.)

Теперь составляем плоскость ( BE{{D}_{1}}.)

( Bleft( 0,0,0 right),~Eleft( 1,0,2 right),~{{D}_{1}}left( 1,1,5 right).)

Cразу же составляем уравнение плоскости:

( left| {begin{array}{*{20}{c}}x&1&1\y&0&1\z&2&5end{array}} right| = 0)

( begin{array}{l}2y+z-2x-5y=0\-2x-3y+z=0\2x+3y-z=0end{array})

Ищем угол:

( cosvarphi =frac{1}{sqrt{4+9+1}}=frac{1}{sqrt{14}})

Ответ: ( arccos frac{1}{sqrt{14}}~~)

Теперь ответы к последним двум задачам:

4. ( arccosfrac{2}{3})

5. ( sqrt{frac{2}{3}})

Ну что же, теперь самое время немного передохнуть, ведь мы с тобой молодцы и проделали огромную работу!

Вычисление расстояния от точки до плоскости

Что нам потребуется для решения этой задачи?

  • Координаты точки ( Mleft( {{x}_{0}},{{y}_{0}},{{z}_{0}} right).)
  • Уравнение плоскости ( Ax+By+Cz+D=0.)

Итак, как только мы получим все необходимые данные, то применяем формулу:

( d=frac{left| A{{x}_{0}}+B{{y}_{0}}+C{{z}_{0}}+D right|}{sqrt{{{A}^{2}}+{{B}^{2}}+{{C}^{2}}}})

Как мы строим уравнение плоскости тебе уже должно быть известно из предыдущих задач, которые я разбирал в прошлой части. Давай сразу приступим к задачам.

Схема следующая: 1, 2 –я помогаю тебе решать, причем довольно подробно, 3, 4 – только ответ, решение ты проводишь сам и сравниваешь. Начали!

Решения:

1. Рисуем кубик с единичными ребрами, строим отрезок и плоскость, середину отрезка ( B{{C}_{1}}) обозначим буквой ( M)

Вначале давай начнем с легкого: найдем координаты точки ( displaystyle M). Так как ( displaystyle Bleft( 0,1,0 right),~{{C}_{1}}left( 1,1,1 right),~) то ( displaystyle Mleft( frac{1}{2},1,frac{1}{2} right).) (вспомни координаты середины отрезка!)

Теперь составляем уравнение плоскости по трем точкам ( displaystyle Aleft( 0,0,0 right),~{{B}_{1}}left( 0,1,1 right),~{{D}_{1}}left( 1,0,1 right).)

(left| {begin{array}{*{20}{c}}x&0&1\y&1&0\z&1&1end{array}} right| = 0)

( displaystyle x+y-z=0.)

( displaystyle A=1,B=1,C=-1,~D=0.)

Теперь я могу приступать к поиску расстояния:

( displaystyle d=frac{left| frac{1}{2}+1-frac{1}{2} right|}{sqrt{1+1+1}}=frac{1}{sqrt{3}})

Ответ: ( displaystyle frac{1}{sqrt{3}})

2. Вновь начинаем с чертежа, на котором отмечаем все данные!

Для пирамиды было бы полезно отдельно рисовать ее основание.

Даже тот факт, что я рисую как курица лапой, не помешает нам с легкостью решить эту задачу!

1. ( AO=OC=frac{1}{2}AC=frac{sqrt{{{2}^{2}}+{{2}^{2}}}}{2}=sqrt{2}).

Тогда ( OS=sqrt{S{{C}^{2}}-O{{C}^{2}}}=sqrt{3}.)

Теперь легко найти координаты точки ( S.)

Так как координаты точки ( O:Oleft( 1,1,0 right),~), то ( Sleft( 1,1,sqrt{3} right).)

2. Так как координаты точки ( C:) ( Cleft( 2,2,0 right),) а ( M) – середина отрезка ( SC), то

( Mleft( frac{3}{2},frac{3}{2},frac{sqrt{3}}{2} right).)

Без проблем найдем и координаты еще двух точек на плоскости ( ADM.) ( Dleft( 1,0,0 right),~Aleft( 0,0,0 right).) Составляем уравнение плоскости и упростим его:

(left| {left| {begin{array}{*{20}{c}}x&1&{frac{3}{2}}\y&0&{frac{3}{2}}\z&0&{frac{{sqrt 3 }}{2}}end{array}} right|} right| = 0)

( frac{3}{2}z-frac{sqrt{3}}{2}y=0)

( sqrt{3}y-3z=0)

( y-sqrt{3}z=0.)

Так как точка ( B) имеет координаты: ( Bleft( 0,2,0 right)), то вычисляем расстояние:

( d=frac{2}{sqrt{1+3}}=1.)

Ответ (очень редкий!): ( 1)

Ну что, разобрался?

Мне кажется, что здесь все так же технично, как и в тех примерах, что мы рассматривали с тобой в предыдущей части. Так что я уверен, что если ты овладел тем материалом, то тебе не составит труда решить оставшиеся две задачи.

Я лишь приведу ответы:

  • ( frac{3sqrt{39}}{4})
  • ( frac{sqrt{3}}{2})

Вычисление расстояния от прямой до плоскости

На самом деле, здесь нет ничего нового. Как могут располагаться прямая и плоскость друг относительно друга?

У них есть всего ( 2) возможности: пересечься, или прямая параллельна плоскости. Как ты думаешь, чем равно расстояние от прямой до плоскости, с которой данная прямая пересекается?

Мне кажется, что тут ясно, что такое расстояние равно нулю. Неинтересный случай.

Второй случай хитрее: тут уже расстояние ненулевое. Однако, так как прямая параллельна плоскости, то каждая точка прямой равноудалена от этой плоскости:

Таким образом:

Расстояние от плоскости до параллельной ей прямой равно расстоянию от любой точки прямой до плоскости.

А это значит, что моя задача свелась к предыдущей: ищем координаты любой точки на прямой, ищем уравнение плоскости, вычисляем расстояние от точки до плоскости.

На самом деле, такие задачи в ЕГЭ встречаются крайне редко. Мне удалось найти лишь одну задачу, и то данные в ней были такими, что метод координат к ней был не очень-то и применим!

Теперь перейдем к другому, гораздо более важному классу задач:

Вычисление расстояния точки до прямой

Что нам потребуется?

  • Координаты точки, от которой мы ищем расстояние: ( {{M}_{0}}left( {{x}_{0}},{{y}_{0}},{{z}_{0}} right).)
  • Координаты любой точки, лежащей на прямой ( {{M}_{1}}left( {{x}_{1}},{{y}_{1}},{{z}_{1}} right))
  • Координаты направляющего вектора прямой ( vec{s}left( m,n,p right))

Какую применяем формулу?

Ответ: ( d=frac{left| overrightarrow{{{M}_{0}}{{M}_{1}}}times vec{s} right|}{sqrt{{{m}^{2}}+{{n}^{2}}+{{p}^{2}}}})

Что означает знаменатель данной дроби тебе и так должно быть ясно: это длина направляющего вектора прямой. Здесь очень хитрый числитель!

Выражение ( left| overrightarrow{{{M}_{0}}{{M}_{1}}}times vec{s} right|) означает модуль (длина) векторного произведения векторов ( overrightarrow{{{M}_{0}}{{M}_{1}}}) и ( vec{s}.)

Как вычислять векторное произведение, мы с тобой изучали в предыдущей части работы. Освежи свои знания, нам они сейчас очень пригодятся!

Таким образом, алгоритм решения задач будет следующий:

  • Ищем координаты точки, от которой мы ищем расстояние: ( {{M}_{0}}left( {{x}_{0}},{{y}_{0}},{{z}_{0}} right).)
  • Ищем координаты любой точки на прямой, до которой мы ищем расстояние: ( {{M}_{1}}left( {{x}_{1}},{{y}_{1}},{{z}_{1}} right))
  • Строим вектор ( overrightarrow{{{M}_{0}}{{M}_{1}}}:) ( overrightarrow{{{M}_{0}}{{M}_{1}}}left( {{x}_{1}}-{{x}_{0}},{{y}_{1}}-{{y}_{0}},{{z}_{1}}-{{z}_{0}} right).)
  • Строим направляющий вектор прямой ( vec{s}left( m,n,p right))
  • Вычисляем векторное произведение ( overrightarrow{{{M}_{0}}{{M}_{1}}}times vec{s})
  • Ищем длину полученного вектора: ( left| overrightarrow{{{M}_{0}}{{M}_{1}}}times vec{s} right|)
  • Вычисляем расстояние: ( d=frac{left| overrightarrow{{{M}_{0}}{{M}_{1}}}times vec{s} right|}{sqrt{{{m}^{2}}+{{n}^{2}}+{{p}^{2}}}})

Работы у нас много, а примеры будут достаточно сложными! Так что теперь сосредоточь все внимание!

1. Дана пра­виль­ная тре­уголь­ная пи­ра­ми­да ( DABC) с вер­ши­ной ( D). Сто­ро­на ос­но­ва­ния пи­ра­ми­ды равна ( sqrt{6}), вы­со­та равна ( sqrt{30}).

Най­ди­те рас­сто­я­ние от се­ре­ди­ны бо­ко­во­го ребра ( BD) до пря­мой ( MT), где точки ( M) и ( T) — се­ре­ди­ны ребер ( AC) и ( AB) со­от­вет­ствен­но.

2. Длины ребер ( AB,A{{A}_{1}}) и ( AD) пря­мо­уголь­но­го па­рал­ле­ле­пи­пе­да ( ABCD{{A}_{1}}{{B}_{1}}{{C}_{1}}{{D}_{1}}) равны со­от­вет­ствен­но ( 12,text{ }16~) и ( 15.)

Най­ди­те рас­сто­я­ние от вер­ши­ны ( {{A}_{1}}) до пря­мой ( B{{D}_{1}}.)

3. В пра­виль­ной ше­сти­уголь­ной приз­ме ( ABCDEF{{A}_{1}}{{B}_{1}}{{C}_{1}}{{D}_{1}}{{E}_{1}}{{F}_{1}}) все ребра ко­то­рой равны ( 1) най­ди­те рас­сто­я­ние от точки ( B) до пря­мой ( {{E}_{1}}{{F}_{1}}.)

Решения:

1. Делаем аккуратный чертеж, на котором отмечаем все данные:

Ну что же, работы нам предстоит немало! Принимаемся за нее, засучив рукава!

1. Чтобы найти координаты высоты пирамиды, нам нужно знать координаты точки ( displaystyle O.) Её аппликата равна нулю, а ордината равна ( displaystyle frac{sqrt{6}}{2}.)

Абсцисса ее равна длине отрезка ( displaystyle OS.) ( displaystyle AS=sqrt{A{{B}^{2}}-S{{B}^{2}}}=sqrt{6-frac{6}{4}}=frac{3}{sqrt{2}}.~)

Так как ( displaystyle AS) – высота равностороннего треугольника ( displaystyle ABC), то она делится в отношении ( displaystyle 2:1), считая от вершины, отсюда ( displaystyle OS=frac{3}{3sqrt{2}}=frac{1}{sqrt{2}}).

Окончательно, получили координаты:

( displaystyle Oleft( frac{1}{sqrt{2}},frac{sqrt{6}}{2},0 right).)

Тогда ( displaystyle D(left( frac{1}{sqrt{2}},frac{sqrt{6}}{2},sqrt{30} right)).

Координаты точки ( displaystyle A:Aleft( frac{3}{sqrt{2}},frac{sqrt{6}}{2},0 right).)

2. ( displaystyle K) – середина отрезка ( displaystyle BD:)

( displaystyle Kleft( frac{1}{2sqrt{2}},frac{sqrt{6}}{4},frac{sqrt{30}}{2} right).~)

3. ( displaystyle M) – середина отрезка ( displaystyle AC:)

( displaystyle Mleft( frac{3}{2sqrt{2}},~frac{frac{sqrt{6}}{2}+sqrt{6}}{2},0 right)=Mleft( frac{3}{2sqrt{2}},~frac{3sqrt{6}}{4},0 right).)

( displaystyle T) – середина отрезка ( displaystyle AB)

( displaystyle Tleft( frac{3}{2sqrt{2}},~frac{sqrt{6}}{4},0 right).~)

4. Координаты( displaystyle overrightarrow{KT}:overrightarrow{KT}left( frac{3}{2sqrt{2}}-frac{1}{2sqrt{2}},frac{sqrt{6}}{4}-frac{sqrt{6}}{4},~0-frac{sqrt{30}}{2} right)=overrightarrow{KT}left( frac{1}{sqrt{2}},~0,~-frac{sqrt{30}}{2} right).)

Координаты вектора ( displaystyle overrightarrow{TM}:)

( displaystyle overrightarrow{TM}left( 0,frac{3sqrt{6}}{4}-frac{sqrt{6}}{4},0 right)=overrightarrow{TM}left( 0,~frac{sqrt{6}}{2},0 right).)

5. Вычисляем векторное произведение:

( displaystyle overrightarrow{KT}times overrightarrow{TM}=frac{1}{sqrt{2}}cdot frac{sqrt{6}}{2}cdot overrightarrow{k}-frac{sqrt{30}}{2}cdot frac{sqrt{6}}{2}cdot vec{i}=frac{3sqrt{5}}{2}vec{i}+frac{sqrt{3}}{2}overrightarrow{k}=left( frac{3sqrt{5}}{2},0,~frac{sqrt{3}}{2} right).)

6. Длина вектора ( displaystyle TM): проще всего заменить, что отрезок ( displaystyle TM) – средняя линия треугольника ( displaystyle ABC), а значит, он равен половине основания ( displaystyle BC). Так что ( displaystyle left| text{ }!!~!!text{ }overrightarrow{TM} right|=frac{sqrt{6}}{2}).

7. Считаем длину векторного произведения:

( displaystyle left| overrightarrow{KT}times overrightarrow{TM} right|=sqrt{{{left( frac{3sqrt{5}}{2} right)}^{2}}+{{left( frac{sqrt{3}}{2} right)}^{2}}}=2sqrt{3}.)

8. Наконец, находим расстояние:

( displaystyle d=frac{left| overrightarrow{KT}times overrightarrow{TM} right|}{text{ }!!~!!text{ }left| text{ }!!~!!text{ }overrightarrow{TM} right|}=frac{2sqrt{3}}{frac{sqrt{6}}{2}}=2sqrt{2})

Уф, ну все!

Честно тебе скажу: решение этой задачи традиционными методами (через построения), было бы намного быстрее.

Зато здесь я все свел к готовому алгоритму!

Я так думаю, что алгоритм решения тебе ясен? Поэтому попрошу тебя решить оставшиеся две задачи самостоятельно. Сравним ответы?

2. ( displaystyle 12)

3. ( displaystyle 2)

Опять-таки повторюсь: эти задачи проще (быстрее) решать через построения, а не прибегая к координатному методу.

Я продемонстрировал такой способ решения лишь затем, чтобы показать тебе универсальный метод, который позволяет «ничего не достраивать».

Наконец, рассмотрим последний класс задач: Вычисление расстояния между скрещивающимися прямыми.

Вычисление расстояния между скрещивающимися прямыми

Здесь алгоритм решения задач будет схож с предыдущим. Что у нас есть:

  • Направляющий вектор первой прямой: ( overrightarrow{{{a}_{1}}(}{{x}_{1}},{{y}_{1}},{{z}_{1}}).)
  • Направляющий вектор второй прямой: ( overrightarrow{{{a}_{2}}(}{{x}_{2}},{{y}_{2}},{{z}_{2}}).)
  • Любой вектор, соединяющий точки первой и второй прямой: ( overrightarrow{{{a}_{3}}}left( {{x}_{0}},{{y}_{0}},{{z}_{0}} right))

Как мы ищем расстояние между прямыми?

Формула следующая:

( d=frac{left| left( overrightarrow{{{a}_{3}}},~overrightarrow{{{a}_{1}}},overrightarrow{{{a}_{2}}} right) right|}{left| overrightarrow{{{a}_{1}}}times overrightarrow{{{a}_{2}}} right|})

Числитель – это модуль смешанного произведения (мы его вводили в предыдущей части), а знаменатель – как и в предыдущей формуле (модуль векторного произведения направляющих векторов прямых, расстояние между которыми мы с тобой ищем).

Я напомню тебе, что

тогда формулу для расстояния можно переписать в виде:

[d = frac{{left| begin{array}{l}begin{array}{*{20}{c}}{{x_0}}&{{y_0}}&{{z_0}}end{array}\begin{array}{*{20}{c}}{{x_1}}&{{y_1}}&{{z_1}}end{array}\begin{array}{*{20}{c}}{{x_2}}&{{y_2}}&{{z_2}}end{array}end{array} right|}}{{left| begin{array}{l}begin{array}{*{20}{c}}{overrightarrow i }&{overrightarrow j }&{overrightarrow k }end{array}\begin{array}{*{20}{c}}{{x_1}}&{{y_1}}&{{z_1}}end{array}\begin{array}{*{20}{c}}{{x_2}}&{{y_2}}&{{z_2}}end{array}end{array} right|}}]

Этакий определитель делить на определитель! Хотя, если честно, мне здесь совсем не до шуток!

Данная формула, на самом деле, очень громоздка и приводит к достаточно сложным вычислениям. На твоем месте я бы прибегал к ней только в самом крайнем случае!

Давай попробуем решить несколько задач, используя изложенный выше метод:

  • В пра­виль­ной тре­уголь­ной приз­ме ( ABC{{A}_{1}}{{B}_{1}}{{C}_{1}}), все рёбра ко­то­рой равны ( 1), най­ди­те рас­сто­я­ние между пря­мы­ми ( A{{A}_{1}}) и ( B{{C}_{1}}).
  • Дана пра­виль­ная тре­уголь­ная приз­ма ( ABC{{A}_{1}}{{B}_{1}}{{C}_{1}}) все рёбра ос­но­ва­ния ко­то­рой равны ( 2sqrt{7}) Се­че­ние, про­хо­дя­щее через бо­ко­вое ребро ( A{{A}_{1}}) и се­ре­ди­ну ( M) ребра ( {{B}_{1}}{{C}_{1}}) яв­ля­ет­ся квад­ра­том. Най­ди­те рас­сто­я­ние между пря­мы­ми ( {{A}_{1}}B) и ( AM.)

Первую решаю я, а опираясь на нее, вторую решаешь ты!

1. Рисую призму и отмечаю прямые ( A{{A}_{1}}) и ( B{{C}_{1}}.)

Координаты точки С: ( C:Cleft( frac{sqrt{3}}{2},frac{1}{2},0 right),) тогда ( {{C}_{1}}left( frac{sqrt{3}}{2},frac{1}{2},1 right).~)

Координаты точки ( B:Bleft( 0,1,0 right).~)

Координаты вектора ( overrightarrow{B{{C}_{1}}}:~overrightarrow{B{{C}_{1}}}left( frac{sqrt{3}}{2},-frac{1}{2},1 right).)

Координаты точки ( {{A}_{1}}:{{A}_{1}}left( 0,0,1 right).)

Координаты вектора ( overrightarrow{A{{A}_{1}}}:~overrightarrow{A{{A}_{1}}}left( 0,0,1 right).)

Координаты вектора ( overrightarrow{AB}left( 0,1,0 right).)

[left( {B,overrightarrow {A{A_1}} overrightarrow {B{C_1}} } right) = left| {begin{array}{*{20}{l}}{begin{array}{*{20}{c}}0&1&0end{array}}\{begin{array}{*{20}{c}}0&0&1end{array}}\{begin{array}{*{20}{c}}{frac{{sqrt 3 }}{2}}&{ — frac{1}{2}}&1end{array}}end{array}} right| = frac{{sqrt 3 }}{2}]

Считаем векторное произведение между векторами ( AA) и ( overrightarrow{B{{C}_{1}}}:)

[overrightarrow {A{A_1}} cdot overrightarrow {B{C_1}} = left| begin{array}{l}begin{array}{*{20}{c}}{overrightarrow i }&{overrightarrow j }&{overrightarrow k }end{array}\begin{array}{*{20}{c}}0&0&1end{array}\begin{array}{*{20}{c}}{frac{{sqrt 3 }}{2}}&{ — frac{1}{2}}&1end{array}end{array} right| — frac{{sqrt 3 }}{2}overrightarrow k + frac{1}{2}overrightarrow i ]

Теперь считаем его длину:

( left| overrightarrow{A{{A}_{1}}}times overrightarrow{B{{C}_{1}}} right|=sqrt{{{left( -frac{sqrt{3}}{2} right)}^{2}}+{{left( frac{1}{2} right)}^{2}}}=1)

Тогда

( d=frac{frac{sqrt{3}}{2}}{1}=frac{sqrt{3}}{2}.)

Ответ: ( frac{sqrt{3}}{2}.)

Теперь постарайся аккуратно выполнить вторую задачу. Ответом на нее будет: ( frac{sqrt{6}}{2}).

Понравилась статья? Поделить с друзьями:
  • Как найти свою аудиторию таргет
  • Как составить типовой договор образец
  • Как найти корабль кальмар no mans sky
  • Как составить открытое письмо с подписями
  • Как найти производную функции многочлена